0% found this document useful (0 votes)
277 views

2019 Explanation k2

The patient, a 35-year-old man, presents with signs of anemia including fatigue, palpitations and pale skin. His medical history includes peptic ulcer disease. A physical exam finds a rapid heart rate and systolic murmur. Blood work shows low red blood cell and hemoglobin levels. Given his history of peptic ulcer disease and signs pointing to blood loss, posthemorrhagic anemia from an ulcer bleeding is the most likely type of anemia in this case.

Uploaded by

berdaderaga
Copyright
© © All Rights Reserved
Available Formats
Download as PDF, TXT or read online on Scribd
0% found this document useful (0 votes)
277 views

2019 Explanation k2

The patient, a 35-year-old man, presents with signs of anemia including fatigue, palpitations and pale skin. His medical history includes peptic ulcer disease. A physical exam finds a rapid heart rate and systolic murmur. Blood work shows low red blood cell and hemoglobin levels. Given his history of peptic ulcer disease and signs pointing to blood loss, posthemorrhagic anemia from an ulcer bleeding is the most likely type of anemia in this case.

Uploaded by

berdaderaga
Copyright
© © All Rights Reserved
Available Formats
Download as PDF, TXT or read online on Scribd
You are on page 1/ 261

2019 – Eng- med –k2

1. A 25-year-old woman has been suffering from diabetes Objective


mellitus since she was 9. She was admitted into the nephrology When the kidney glomerular filtration rate (GFR)
unit with significant edemas of the face, arms, and legs. Blood falls below 15 mL/min the patient is considered
pressure - 200/110 mm Hg, Hb- 90 g/L, blood creatinine - 850 end-stage kidney disease (stage 5) and indicated
mcmol/L, urine proteins - 1.0 g/L, leukocytes - 10-15 in the for hemodialysis.
vision field. Glomerular filtration rate 10 mL/min. What tactics
should the doctor choose?
A. Transfer into the hemodialysis unit
B. Active conservative therapy for diabetic nephropathy
C. Dietotherapy
D. Transfer into the endocrinology clinic
E. Renal transplantation
Related
A 23-year-old man has accidentally swallowed brake fluid.
Signs of kidney damage (↓ GFR )
After that he has been presenting with anuria for 5 days already; 1. ↑ Creatinine: > 0.106 μmol/L (106 mmol/L).
his creatinine levels elevated up to 0.569 mmol/L. What 2. Hyperkalaemia > 5mmol/L
treatment tactics should be chosen in this case? 3. High BUN: > 15
A. Hemodialysis
B. Detoxication therapy
C. Antidotal therapy
D. Diuretics Indications for dialysis
E. Plasmapheresis 1. Acute or chronic kidney injury: ↓ GFR
(Anuria) = ↑ toxins (↑ creatinine indicator).
A 30-year-old woman with a long history of chronic 2. Drug overdose or toxicity: ethylene glycol
pyelonephritis complains of considerable weakness, sleepiness, (break fluid)
decrease in diuresis down to 100 ml per day. BP is 200/120 mm 3. Fluid overload: Dialysis may be necessary to
Hg. In blood: creatinine - 0,62 millimole/l, hypoproteinemia, remove excess fluid.
albumines - 32 g/l, potassium - 6,8 millimole/l, hypochromic 4. Electrolyte imbalances: Dialysis may be
anemia, increased ESR. What is the first step in the patient necessary to correct electrolyte imbalances,
treatment tactics? such as high potassium levels, that can occur
A. Haemodialysis when the kidneys are not functioning
B. Antibacterial therapy properly.
C. Enterosorption
D. Haemosorption
E. Blood transfusion
2. A 59-year-old woman was brought into the
rheumatology unit. Extremely severe case of
scleroderma is suspected. Objectively she presents
with malnourishment,” mask-like” face, and acro-
osteolysis. Blood: erythrocytes - 2.2 · 109/L,
erythrocyte sedimentation rate 40 mm/hour. Urine:
elevated levels of free oxyproline. Name one of the
most likely pathogenetic links in this case:
A. Formation of antibodies to collagen
B. Formation of antibodies to native DNA
C. Formation of antibodies to blood corpuscles
D. Formation of antibodies to transversely striated
muscles
E. Formation of antibodies to vessel wall

Objective
Scleroderma is an autoimmune disease
characterized by the formation of activating
antibodies leading to overproduction of collagen.
2013
A 28-year-old female patient with a six-year history
of Raynaud’s syndrome has recently developed pain
in the small joints of hands, difficult movement of
food down the esophagus. What kind of disease can
you think of in this case?
A. Systemic scleroderma
B. Periarteritis nodosa
C. Rheumatoid arthritis
D. Systemic lupus erythematosus
E. Pseudotrichiniasis

Objective
GIT and joint involvment of systmeic scleroderma
3. A 34-year-old man on the 3rd day of ceftriaxone treatment for Objective
acute otitis (daily dosage - 2 grams) developed diarrhea In this patient, wide-spectrum ceftriaxone
occurring 5-6 times per day. Feces are without mucus or blood antibiotic is used to kill the bacteria causing the
admixtures. Temperature is 36.6oC. Gregersen reaction (occult acute otitis, but it has also killed the normal
blood in feces) is negative. Stool culture detected no pathogenic bacterial in the intestine leading to dysbiosis and
germs. What is the most likely cause of diarrhea in this case? diarrhea.
A. Antibiotic-associated diarrhea
B. Intestinal dysbiosis
C. Bacterial overgrowth syndrome
D. Ulcerative colitis
E. Crohn’s disease (regional enteritis)
Details

Overview:
Antibiotic-associated diarrhea (AAD) is a common side effect of taking
antibiotics, which are medications used to treat bacterial infections. It is
estimated that up to 25% of people taking antibiotics may develop AAD. It
occurs when the normal balance of bacteria in the intestines is disrupted by
antibiotics, which can kill both harmful and beneficial bacteria. This
disruption can allow harmful bacteria such as Clostridium difficile (C.
difficile) to overgrow and produce toxins, leading to diarrhea.

Prevention and treatment of AAD includes:


1. Taking antibiotics only when necessary.
2. Taking probiotics, which are live bacteria that can help restore the
balance of beneficial bacteria in the intestines.
3. Drinking plenty of fluids to prevent dehydration.
4. If AAD does occur, treatment may involve stopping the antibiotic,
switching to a different antibiotic, or taking medication to treat the
infection and diarrhea.
4. A chronic alcoholic was hospitalized into the therapeutic Objective
inpatient unit due to pneumonia. On the day 5 of his the long hospitalization of the alcoholic patients
hospitalization, he became disoriented in time and space, can lead to the development of alcohol
developed fear-inducing visual hallucinations and motor withdrawal syndrome symptoms.
agitation. Full body tremor and tremor of the limbs are
observed. X-ray and physical examinations detect the signs of
his convalescence from pneumonia. What tactics should be
chosen regarding this patient?
A. Transfer into the inpatient narcology department
B. Continue the treatment in the therapeutic department
C. Transfer into the neuroresuscitation department
D. Compulsory medical treatment for alcoholism
E. Discharge from the hospital
5. After eating shrimps, a 25-year-old man suddenly developed Objective
skin itching, some areas of his skin became hyperemic or Acute urticaria is a skin allergic reaction that can
erupted into vesicles. Make the diagnosis: occur to certain type of foods like shrimps in this
A. Acute urticaria case.
B. Hemorrhagic vasculitis (Henoch-Schonlein
C. purpura)
D. Urticaria pigmentosa
E. Psoriasis
F. Scabies
Related

A woman with atopic bronchial asthma was found to have one


allergen to dog hair +++. Carpets were removed from the
apartment, the apartment was renovated, and air conditioner was
installed. However, recurrent asphyxia attacks still occur every
night, despite the patient undergoing pathogenetic therapy. What
long-term treatment tactics can help this patient to decrease her
sensitivity to the allergen?
A. Continuation of prior treatment
B. Antihistamine therapy
C. Specific hyposensitization
D. Buteyko breathing technique
E. Referral for speleotherapy
Objective
6. A 25-year-old woman complains of fatigue, dizziness,
• All blood cells in this patient is low which
hemorrhagic rashes on the skin. She has been presenting with
means she is suffering from pancytopenia.
these signs for a month. Blood test: erythrocytes - 1.0 1012/L, • Pancytopenia is a deficiency of all three types
Hb- 37 g/L, color index - 1.1, leukocytes -1.2·109/L, platelets - of blood cells: red blood cells, white blood
42·109/L. What analysis would be the most advisable for cells, and platelets.
diagnosis making in this case? • The location of blood production
A. Sternal puncture (bone marrow biopsy) (hematopoiesis) is bone marrow.
B. Splenic biopsy • A bone marrow biopsy can help to identify the
C. Liver biopsy underlying cause of pancytopenia, such as a
D. Coagulation studies bone marrow disorder, infection, or exposure
E. US of the gastrointestinal tract to certain medications or toxins.
• The biopsy is taken from hipbone or sternum.
7. A 35-year-old man complains of rapidly incresing fatigue, Objective
palpitations,” visual snow”, and dizziness. He has a history of Peptic ulcer disease →
peptic ulcer of the stomach. Objectively the skin is pale. Hemorrhage →
Vesicular respiration is observed in the lungs. Systolic murmur Loss of blood cells →
is detected over the cardiac apex, heart rate is 100/min., BP is Posthemorrhagic anemia
100/70 mm Hg. The epigastrium is slightly tender on palpation.
Blood test: erythrocytes - 3.2 · 1012/L, Нb- 100 g/L, color index
0.95. What type of anemia is the most likely present in this
case?
A. Posthemorrhagic anemia
B. Sideroblastic anemia
C. Chronic iron-deficiency anemia
D. Hemolytic anemia
E. Hypoplastic anemia
8. A 62-year-old patient has been hospitalized with complaints
of pain in the thorax on the right during breathing, dyspnea, and Objective
dry cough. Ten days ago, he slipped and fell hitting his right Fracture of the III-IV ribs →
side. On examintaion: the patient lies on the left side. The right Thoracic surgery department
side of the thorax lags during breathing. On the right there are
crepitation and pain in the III-IV ribs. Dullness of percussion
sound and sharply diminished breath sounds can be observed.
On X-ray: signs of exudate, fracture of the III-IV ribs. On
pleurocentesis: blood is detected. Choose the further tactics:
A. Transfer to a thoracic surgery department
B. Prescribe conservative therapy
C. Perform repeated pleural taps
D. Apply a fixation bandage to the rib cage
E. Refer to a traumatologist
9. 51-year-old man complains of vomiting with blood. He has Objective
been drinking alcohol excessively. Health disorder has been • Drinking alcohol excessively →
observed since he was 40, when he first developed jaundice. On • Liver cirrhosis →
examination the skin and visible mucosa are icteric, with a • Liver failure →
stellate vascular pattern. The patient is malnourished and • Can’t metabolize bilirubin → Jaundice.
presents with abdominal distension, umbilical hernia, and • Can’t pass blood → portal hypertension →
ascites. The edge of the liver is tapered and painless, +3 cm, the Ascites.
spleen is +2 cm. Blood test: Hb- 80 g/L, leukocytes - 3·109/L, • Can’t make proteins → skin changes
platelets - 85·109/L. What is the cause of portal hypertension in
this patient?
A. Hepatic cirrhosis
B. Thrombosis of the splenic vein
C. Hemochromatosis
D. Constrictive pericarditis
E. Budd-Chiari syndrome
10. A young man has made an appointment with the Objective
dermatologist. He complains of a painful facial rash in the beard Sycosis is an inflammatory reaction to hair
and mustache area. This condition has been persisting for penetrating the skin characterized by firm papules
several weaks already. After shaving, the patient’s condition and pustules.
aggravates. The diagnosis of sycosis is made. What primary
morphological elements can be observed in the rash in this
case?
A. Pustules, papulae
B. Nodes, nodules
C. Pustules, bumps
D. Phlyctenae, maculae
E. Maculae, nodes
11. At night a 63-year-old woman suddenly developed an Objective
asphyxia attack. She has a 15year-long history of essential • This patient is suffering from hypertensive
hypertension and had a myocardial infarction 2 years ago. crisis (> 180/120 mmHg).
Objectively her position in bed is orthopneic, the skin is pale, • One of the complications of hypertensive
the patient is covered with cold sweat, acrocyanosis is observed. crisis is left ventricular heart failure.
Pulse - 104/min. Blood pressure - 210/130 mm Hg, respiration
rate - 38/min. Pulmonary percussion sound is clear, with slight
dullness in the lower segments; throughout the lungs single dry
crackles can be heard that become bubbling and non-resonant in
the lower segments. What is the most likely complication in this
patient?
A. Acute left ventricular failure
B. Paroxysmal tachycardia
C. Bronchial asthma attack
D. Pulmonary embolism
E. Acute right ventricular failure
12. A 26-year-old woman is suspected to suffer from systemic
lupus erythematosus due to systemic lesions of skin, vessels, Objective
joints, serous tunics, and heart that developed after Presence of DNA antibodies is most important
photosensitization. The following is detected: LE cells, specific indicators for the diagnosis of lupus.
antibodies to native DNA, isolated anti-centromere antibodies,
rheumatoid factor is 1:100, Wassermann reaction is positive,
circulating immune complex is 120 units. What immunological
indicators are considered to be specific to this disease?
A. DNA antibodies
B. Rheumatoid factor
C. Anti-centromere antibodies
D. Immunoglobulin A
E. Increased circulating immune complex
A 30-year-old patient presented with body temperature rise up to 38,
5oC, pain in the small articulations of hands; face edemata and erythema.
In blood: RBCs - 2, 6 · 1012/l; Hb- 98 %/&; WBCs - 2 · 109/l; ESR - 58
mm/h. In the urine: protein - 3,1 g/l; RBCs - 10-15 in the vision field.
What disease can be suspected in this case?
A. Systemic lupus erythematosus
B. Sepsis
C. Systemic scleroderma
D. Periarteritis nodosa
E. Acute glomerulonephritis

A 34-year-old patient after vacation in Crimea has developed severe pain


in her elbow joints, dyspnea and weakness. Body temperature is 37, 6oC,
skin pallor and erythema on the cheeks and bridge of nose are observed,
lip mucosa is ulcerated. The joints are not visibly deformed, movement
of the right elbow joint is restricted. Pleura friction sound is detected in
the lungs on the right side below the angle of scapula. Heart sounds are
dull, tachycardia, gallop rhythm, heart rate - 114/min, BP - 100/60 mm
Hg. The most likely diagnosis is:
A. Systemic lupus erythematosus
B. Rheumatic carditis
C. Rheumatoid arthritis
D. Infectious allergic myocarditis
E. Dry pleurisy
13. A woman came to the doctor with complaints of increased Objective
body temperature up to 37.8oC and moderately sore throat for • Tonsils are hypertrophied and covered with
the last 3 days. Objectively: mandibular lymph nodes are gray coating are typical finding for diphtheria
enlarged up to 3 cm. Palatine tonsils are hypertrophied and infection.
covered with gray coating that spreads to the uvula and anterior • Note: candidiasis will cause white coating in
pillars of the fauces. What is the most likely diagnosis? the tongue not tonsils
A. Oropharyngeal diphtheria
B. Infectious mononucleosis
C. Pseudomembranous (Vincent’s) tonsillitis
D. Agranulocytosis
E. Oropharyngeal candidiasis
14. A 42-year-old man, a dispatcher, suffes from peptic ulcer
disease of the duodenum. The disease is of moderate severity.
He wants to be assigned a disability group. Make the conclusion
regarding his working ability:
A. Capable of working, employable
B. Capable of working, non-employable
C. First group of disability
D. Second group of disability
E. Third group of disability
15. A 58-year-old man complains of weakness and
tumor-like formations that appeared on the anterior
surface of his neck and in the inguinal region.
Palpation detects soft painless mobile cervical and
inguinal lymph nodes up to 2 cm in diameter. The liver
protrudes by 2 cm from the edge of the costal margin,
the lower splenic pole is at the umbilical level. In
blood: erythrocytes - 3.5 · 1012/L, Hb- 88 g/L,
leukocytes - 86 · 109/L, band neutrophils - 1%,
segmented neutrophils - 10%, lymphocytes - 85%,
eosinophils - 2%, basocytes - 0%, monocytes - 2%,
erythrocyte sedimentation rate - 15 mm/hour,
Gumprecht shadows. What is the most likely
diagnosis?
A. Chronic lymphatic leukemia
B. Lymphocytic leukemoid reaction
C. Acute leukemia
D. Chronic myeloleukemia
E. Lymphogranulomatosis
16. A 23-year-old man complains of facial edemas, headache,
dizziness, low urinary output, and urine discoloration (dark red). Objective
These complaints arose after a case of acute tonsillitis. On • Presence of renal
(Glomerulonephritis), heart
examination there are facial edemas, the skin is pale,
symptoms (Rheumatic fever),
temperature is 37.4oC; heart rate is 86/min., blood pressure is
arthritis after pharyngeal infection is
170/110 mm Hg. Heart sounds are muffled, the II heart sound is
a key feature of beta-hemolytic
accentuated over the aorta. What etiological factor is the most streptococcus pyogenes.
likely in this case? • The diagnosis can be made by
A. Beta-hemolytic streptococcus measuring levels of Anti-streptolysin
B. Staphylococcus aureus O.
C. Streptococcus viridans
D. Streptococcus pyogenes
E. Staphylococcus saprophyticus
17. An 18-year-old young man complains of pain in his knee Objective
and elbow joints and body temperature up to 39.5oC. One week • Presence of renal
and a half earlier he developed sore throat. On examination his (Glomerulonephritis), heart
body temperature is 38.5oC. Swelling of the knee and elbow symptoms (Rheumatic fever),
joints is observed. Pulse is 106/min., rhythmic. Blood pressure arthritis after pharyngeal infection is
is 90/60 mm Hg. Cardiac borders are unchanged, heart sounds a key feature of beta-hemolytic
are weakened, at the cardiac apex there is a soft systolic streptococcus pyogenes.
murmur. What factor would be the most indicative of the likely • The diagnosis can be made by
disease etiology? measuring levels of Anti-streptolysin
A. Anti-streptolysin O O.
B. C-reactive protein
C. Creatine kinase
D. Rheumatoid factor
E. Seromucoid

Check the file for more information


18. A woman has been provisionally diagnosed with
Objective
pheochromocytoma. At the stage of intermission her BP is
• Pheochromocytoma is a rare adrenal gland
within norm; there is a tendency towards tachycardia. No urine
tumor that can cause excess release of
pathologies. The decision has been made to perform a catecholamines → blood pressure and other
provocative test with histamine. What drug should be kept close symptoms.
at hand for emergency aid in case of positive test result? • The pheochromocytoma provocative test with
A. Phentolamine histamine/glucagon is a medical diagnostic test
B. Pipolphen (Promethazine) used to detect the presence of
C. Nifedipine pheochromocytoma.
D. Mesaton (Phenylephrine) • During the test, a small dose of histamine (a
E. Prednisolone chemical that can cause dilation of blood
vessels and increased heart rate) is
administered intravenously. If the patient has a
pheochromocytoma, the tumor cells will
release excess amounts of adrenaline and
noradrenaline in response to the histamine,
leading to a marked increase in blood pressure
and heart rate.
• Phentolamine should be present to reverse the
effect of catecholamines in case the patient
developed hypertension.
19. A 40-year-old man with Bekhterev disease (ankylosing
Objective
spondylitis) complains of elevated body temperature up to
• Ankylosing spondylitis (AS) is a type of
37.8oC, back pain and stiffness, especially observed during the inflammatory arthritis that primarily affects
second half of the night. This condition has been lasting for 2 mainly the spine → bamboo spine (vertebral
years. Objectively: reduced spinal mobility, painful sacroiliac fusion).
joint, erythrocyte sedimentation rate - 45 mm/hour. X-ray shows • Associations: patients with AS can also
narrowing of the intervertebral disc space and of the sacroiliac develop inflammation of the uveitis (ex,
joint. What eye pathology is often associated with this type of Iridocyclitis).
disease progression? • Terminology: "Ankylos” means "stiffness" or
A. Iridocyclitis "fusion," and "spondylos" means "vertebra."
B. Retinal detachment
C. Cataract
D. Optic nerve atrophy
E. Blepharitis
20. A 63-year-old man complains of unmotivated
weakness and pressing and bursting sensation in the left
subcostal area. According to him, these signs have been
present for a year already. Previously he was healthy. He
took part in containment measures during the accident at
the Chornobyl Nuclear Power Plant. Objectively: the
skin is pale, peripheral lymph nodes are not enlarged,
the liver is +3 cm, the spleen is +10 cm. Complete blood
count: erythrocytes - 3.1 · 1012/L, Hb- 100 g/L,
leukocytes - 46 · 109/L, blasts - 2%, promyelocytes -
10%, myelocytes 18%, band neutrophils - 27%,
segmented neutrophils - 10%, lymphocytes - 12%,
eosinophils - 6%, basocytes - 3%, monocytes - 2%,
erythrocyte sedimentation rate - 20 mm/hour. What is
the most likely diagnosis?
A. Chronic myeloleukemia
B. Hepatic cirrhosis
C. Acute leukemia
D. Hemolytic anemia
E. Chronic lymphatic leukemia
21. For three years a 31-year-old woman has been complaining
Objective
of pain and swelling of her radiocarpal and
Types of arthritis
metacarpophalangeal articulations and their reduced mobility in
1. Rheumatoid Arthritis:
the morning, which persisted up to 1.5 hours. Two weeks ago
• Erosion (usuration) and joint space
she developed pain, swelling, and reddening of her knee joints, narrowing
her body temperature increased up to 37.5oC. The treatment was 2. Osteoarthritis:
untimely. Examination of the internal organs revealed no • Joint space narrowing
pathologic alterations. Diagnosis of rheumatoid arthritis was • Subchondral sclerosis
made. What changes are most likely to be visible on the
arthrogram?
A. Joint space narrowing, usuration
B. Joint space narrowing, subchondral osteosclerosis
Normal rheumatoid arthritis
C. Cysts in the subchondral bone
D. Numerous marginal osteophytes
E. Epiphyseal osteolysis

Check the video for more details.


Related question

A 10-year-old boy with symptoms of arthritis and myocarditis


was brought to a hospital. Based on clinical examination the
provisional diagnosis of juvenile rheumatoid arthritis was made.
What symptom is the most contributive for the diagnostics of
this disease?
A. Reduced mobility of the joints in the morning
B. Regional hyperemia of the joints
C. Affection of the large joints
D. Enlarged heart
E. Increased heart rate
22. A 52-year-old woman has been suffering for 2 years from
Objective
dull, occasionally exacerbating pain in her right subcostal area,
• Patient presentations is typical for gallbladder
occurring after eating high-fat foods, bitter taste in her mouth in inflammation (cholecystitis).
the morning, constipations, and flatulence. Objectively she has • Ultrasound is the most important diagnostic
excess weight, her body temperature is 36.9oC; there is a device for the diagnosis of cholecystitis.
coating on the root of her tongue; the abdomen is moderately • Why do patients with cholecystitis have pain
distended and painful in the area of gallbladder projection. What after fatty meals?
examination would be the most helpful for diagnosis making?
A. Ultrasound
B. Duodenal intubation
C. Cholecystography
D. Duodenoscopy
E. Liver scanning
23. A 57-year-old woman complains of weakness, dyspnea, loss
Objective
of appetite, and liquid feces. She has been suffering from this
Megaloblastic anemia and b 12 deficiency
condition for 2 years. Objectively she presents with pale skin,
subicteric sclerae, and bright-red fissured tongue. Lymph nodes • Patient findings: ↑ color index > 1 and
macrocytosis is typical for megaloblastic
are not enlarged. Pulse - 100/min. BP- 105/70 mm Hg. Liver +3
anemia.
cm, the spleen cannot be palpated. Blood test: erythrocytes - 1.2
• One of the most important causes of
· 1012/L, Нb- 56 g/L, color index - 1.4, macrocytes, leukocytes - megaloblastic anemia is vitamin b12
2,5 · 109/L, eosinophils - 1%, juvenile - 1%, metamyelocytes - deficiency.
1%, band neutrophils - 8%, segmented neutrophils 47%, • Symptoms of b12 deficiency:
lymphocytes - 38%, monocytes - 4%, reticulocytes - 0.1%, • Megaloblastic anemia: No B12 → No
platelets - 100 · 109/L, ESR- 30 mm/hour, indirect bilirubin - 26 folate activation → No DNA →
mmol/L. What changes can be expected in the bone marrow Megaloblastic anemia.
puncture material? • Neurological symptoms: tingling or
A. Prevalence of megaloblasts numbness.
B. Increased number of sideroblasts • Pale or jaundiced skin: due to decreased
C. Erythroid hyperplasia number of erythrocytes reaching the skin.
D. Presence of blast cells
E. Prevalence of lymphoid tissue
Related questions

After a total gastric resection, the patient developed severe A patient with megaloblastic anemia was taking a water-
B12-deficient anemia with disturbed hematopoiesis. soluble vitamin. Name this substance:
Changed erythrocytes appeared in the patient’s blood. One A. Thiamine chloride
of the signs of this anemia is the presence of the following in B. Tocopherol acetate
blood: C. Ascorbic acid
A. Normocytes D. Cyanocobalamin
B. Elliptocytes E. Pyridoxine
C. Megalocytes
D. Anulocytes A 56-year-old patient came to a hospital with complaints
E. Microcytes about general weakness, tongue pain and burning,
sensation of limb numbness. In the past he underwent
A patient with glossitis presents with disappearance of resection of forestomach. In blood: Hb- 80 g/l; erythrocytes
lingual papillae, reddening and burning pain in the tongue. - 2, 0 1012/l; colour index - 1,2, leukocytes - 3, 5 109/l.
Blood test: erythrocytes - 2.2 · 1012/l, hemoglobin - 103 g/l, What anemia type is it?
color index - 1.4. What type of anaemia is it? A. Hemolytic
A. Iron deficiency B. Posthemorrhagic
B. α-thalassemia C. B12-folate deficient
C. B12 folate-deficiency D. Aplastic
D. β-thalassemia E. Iron-deficient
E. Iron refractory
24. A 35-year-old man suffers from insulindependent diabetes
Objective
mellitus and chronic cholecystitis. He takes NPH insulin: 20
• After eating the meal, the patients started
units in the morning and 12 units in the evening. After a meal he
having the symptoms of hyperglycemia (ex,
developed pain in the right subcostal area, nausea, vomiting,
polyuria).
sleepiness, and increased polyuria. What prehospital measures • Complication of hyperglycemia can be
will be the most effective for prevention of crisis within the next prevented by increasing the dose of insulin.
several hours?
A. Change insulin regimen
B. Take analgesics
C. Take cholagogues
D. Exclude fats from the diet
E. Decrease carbohydrates in the diet
25. A 45-year-old woman complains of intolerable paroxysmal Objective
facial pain on the left with attacks that last for 1-2 minutes. Trigeminal neuralgia
Attacks are provoked by chewing. The disease onset was two • What is it? Recurrent brief episodes of intense
months ago after the overexposure to cold. Objectively: pain at unilateral pain in trigeminal CN V (usually V2
the exit points of the trigeminal nerve on the left. Touching near and/or V3).
the wing of the nose on the left induces a pain attack with tonic • Cause? Most cases are due to compression of
spasm of the facial muscles. What is the most likely diagnosis? CN V root by an aberrant vascular loop.
A. Trigeminal neuralgia • Pain is described as electric shock–like or
B. Glossopharyngeal neuralgia stabbing and usually lasts seconds.
C. Temporomandibular joint arthritis • Typically triggered by light facial touch or
D. Facial migraine facial movements (eg, chewing, talking).
E. Maxillary sinusitis • Treatment: carbamazepine, oxcarbazepine.
Related topic

After a long drive with the window open a man developed facial Facial palsy
asymmetry; he cannot close his right eye, his right nasolabial (Neuropathy of the facial nerve)
fold is smoothed out, movements of expression are absent on • Usually develops after HSV reactivation.
the right, there is a disturbance of gustatory sensation in the • Treatment: glucocorticoids +/– acyclovir. Most
tongue on the right. No other neurological pathologies were patients gradually recover function, but
detected. What disease can be provisionally diagnosed in this aberrant regeneration can occur.
patient? • Other causes of peripheral facial palsy include
A. Neuropathy of the facial nerve Lyme disease, herpes zoster (Ramsay Hunt
B. Neuropathy of the trigeminal nerve syndrome), sarcoidosis, tumors (eg, parotid
C. Trigeminal ganglionitis gland), diabetes mellitus..
D. Neuropathy of the oculomotor nerve
E. Ischemic stroke
Facial palsy vs trigeminal neuropathy

• Facial palsy and trigeminal neuropathy are two distinct


conditions that affect different nerves in the face.
• Facial palsy, also known as facial nerve neuropathy or Bell's
palsy, affects the function of the facial nerve, which controls the
muscles of the face. This can result in weakness or paralysis of
the facial muscles on one side of the face, causing difficulty with
facial expressions, speech, and other functions. The condition is
usually temporary and can be caused by viral infections, trauma,
tumors, or other underlying medical conditions.
• Trigeminal neuropathy, on the other hand, affects the trigeminal
nerve, which provides sensation to the face and controls the
muscles involved in chewing. This can result in pain, numbness,
or tingling in the face, as well as difficulty with chewing and
swallowing. Trigeminal neuropathy can be caused by a variety of
factors, including nerve damage, viral infections, and other
underlying medical conditions.
26. A 28-year-old man complains of skin rash and itching on the
both of his hands. The condition persists for 1.5 years. The Objective
exacerbation of his condition he ascribes to the occupational Allergic contact dermatitis: Type IV
contact with formaldehyde resins. Objectively the lesion foci are hypersensitivity reaction secondary to contact
symmetrically localized on both hands. Against the background allergen (eg, nickel D, poison ivy E , neomycin).
of erythema with blurred margins there are papulae, vesicles,
erosions, crusts, and scales. What is the most likely pathology?
A. Occupational eczema
B. Idiopathic eczema
C. Allergic dermatitis
D. Simple contact dermatitis
E. Erythema multiforme
27. A 20-year-old student after failing an exam developed
Objective
complaints of a sensation of a round foreign body in her throat,
Outdated terms
difficult swallowing. She fixates on her condition, limits her
• Hysterical neurosis: a group of psychological
diet, often cries, seeks attention, exhibits demonstrative attitude.
disorders characterized by physical symptoms
She is highly susceptible to psychotherapeutic suggestion. What
without an apparent physical cause.
psychiatric diagnosis can be made in this case?
• Hypochondriacal neurosis: excessive anxiety
A. Hysterical neurosis
or fear about having a serious illness, despite
B. Hypochondriacal neurosis
having no or only mild physical symptoms
C. Depressive neurosis
• Depressive neurosis: prolonged and persistent
D. Obsessive neurosis
feelings of sadness, hopelessness, and a loss of
E. Paranoid personality disorder
interest in activities that were once pleasurable.
• Obsessive neurosis: persistent and unwanted
thoughts, impulses, or images (obsessions) that
cause significant distress and anxiety
• Paranoid personality disorder: type of
personality disorder characterized by a
pervasive distrust and suspicion of others,
including their motives and intentions.
28. A woman with atopic bronchial asthma
was found to have one allergen to dog hair
+++. Carpets were removed from the
apartment, the apartment was renovated, and
air conditioner was installed. However,
recurrent asphyxia attacks still occur every
night, despite the patient undergoing
pathogenetic therapy. What long-term
treatment tactics can help this patient to
decrease her sensitivity to the allergen?
A. Specific hyposensitization
B. Continuation of prior treatment
C. Antihistamine therapy
D. Buteyko breathing technique
E. Referral for speleotherapy
29. A 20-year-old man was hospitalized on the 9th day of the Objective
disease. He attributes his disease to eating of insufficiently Trichinella
thermally processed pork. At its onset this condition manifested • Trichinella is a parasitic roundworms that
as periorbital edemas and fever. Objectively his body causes edema in face and eyelids.
temperature is 38.5 C. The face is puffy and the eyelids are
• Transmission: pig/pork
markedly swollen. Palpation of gastrocnemius muscles is
sharply painful. Blood test shows hypereosinophilia. What is the
etiology of this disease?
A. Trichinella
B. Trichuris
C. Ascarididae
D. Echinococci
E. Leptospira
30. A 40-year-old man claims that his wife is cheating on him Objective
and presents a” proof” of her infidelity. He repeatedly initiated
scandals with his wife at home and at work, demanding that she
The intervention of the psychiatrist will help
confess her infidelity, insulted her, and threatened to kill her.
preventing dangerous actions by providing proper
What preventive measures should be taken against socially
pharmacological therapy if necessary.
dangerous actions on his part?
A. Consultation with the psychiatrist
B. Outpatient treatment
C. Consultation with the general practitioner
D. Consultation with the psychologist
E. Family counseling
31. A 55-year-old woman complains of pain and popping
sounds in her left knee joint, which occur when she climbs the Check the collection
stairs. Occasionally during movements her joint becomes”
stuck”. 5 years ago, she suffered a trauma of her left knee.
Complete blood count and biochemical blood analysis show
normal results. X-ray shows marked osteosclerosis and
osteophytes. The joint space is narrowed. Make the provisional
diagnosis:
A. Osteoarthritis
B. Rheumatoid arthritis
C. Gouty arthritis
D. Psoriatic arthritis
E. Reactive arthritis
32. A 40-year-old man, a welder, uses manganese electrodes in Objective
his line of work (18 years of experience). He complains of
difficulties with writing, bad mood, inertness, gait • The patient presents with Parkinson's disease
abnormalities, problems with speech, and hand tremors. symptoms such as tremors, stiffness, and
Objectively the following is observed in the patient: hypomimia, difficulty with movement and speech.
increased muscle tone of plastic type, and quiet monotonous • Long-term exposure to high levels of
speech, tremor of the tongue, pill-rolling tremor of the fingers,
manganese may increase the risk of developing
and retropulsion. What syndrome developed in this patient due
Parkinson's disease
to manganese poisoning?
A. Parkinsonism
B. Meningism
C. Hypothalamic syndrome
D. Polyneuritic syndrome
E. Vestibular syndrome
33. A 45-year-old man developed constricting retrosternal pain
that occurs during walks at the distance of 200 m. Objectively: Objective
heart rate is 80/min., BP is 160/90 mm Hg. During Constricting chest pain during at normal activity
cardiopulmonary exercise test at 50 W there is a depression of and S-T segment depression suggest the patient is
S-T segment by 3 mm below the isoline in V3-V4. What is the a class III according to CSS.
provisional diagnosis?
A. Exertional angina pectoris, functional class III
B. Exertional angina pectoris, functional class IV
C. Exertional angina pectoris, functional class II The Canadian Cardiovascular Society (CCS) functional
D. Somatoform autonomic dysfunction, hypertension type classes of angina
E. Alcoholic myocardiodystrophy 1. Class I: mild symptoms and experience angina with
unusually strenuous activity.
2. Class II: mild to moderate symptoms and experience
angina with moderate exertion, such as climbing stairs
or walking uphill.
3. Class III: Patients have moderate to severe symptoms
and experience angina with mild exertion, such as
walking at a normal pace on level ground. S-T segment
depression.
4. Class IV: Patients have severe symptoms and experience
angina at rest or without any physical activity. S-T
segment depression.
34. A 45-year-old man, a farmer, presents with acute onset of a
disease. He complains of headache, high temperature, pain in Objective
the gastrocnemius muscles, icteric face, and dark urine. The patient findings is typical for leptospirosis
infection.
Objectively: body temperature - 38oC, blood pressure - 100/70
mm Hg, conjunctival hemorrhages, hepatosplenomegaly, and
oliguria. What is the most likely provisional diagnosis?
A. Leptospirosis
B. Brucellosis
C. Viral hepatitis
D. Pseudotuberculosis
E. Trichinosis
35. A woman has been working as a polisher for a year and a
half. Her workstation is equipped with a grinding machine Objective
(grinding wheels). She complains of white discoloration of her • Vibration disease is an occupational disease
caused by prolonged and repeated exposure to
fingers and toes that appears when she is nervous. Objectively
vibrating tools or equipment.
there are no changes in the coloration of the distal segments of
• Symptoms include numbness, discoloration,
her limbs. Grip strength measured with a dynamometer is 25 kg,
tingling, pain, and reduced sensitivity in the
algesimetry findings are 0.1; 0.3; 0.5. Cold stimulus is affected areas.
extremely positive on the upper and lower limbs. Internal
organs are without pathologies. Make the diagnosis:
A. Vibration disease
B. Raynaud disease
C. Syringomyelia
D. Raynaud syndrome
E. Polyneuritis
Objective
36. A 37-year-old man suddenly developed acute headache
• Cerebrospinal fluid, which is a clear,
accompanied by nausea, vomiting, and impaired consciousness. colorless liquid that surrounds the brain
Objectively blood pressure is 190/120 mm Hg, the face is and spinal cord.
hyperemic. Patient’s consciousness is clouded, his answers to • The patient is suffering from hypertensive
the questions are short, monosyllabic. Movement and sensory crisis ( > 180/120 mmHg).
disturbances are absent. Meningeal signs are positive. • Hypertensive crisis can lead to serious
Cerebrospinal fluid contains blood. What provisional diagnosis complication including stroke and bleeding
can be made? in the brain and its layers leading to blood
A. Subarachnoid hemorrhage in the CSF (Subarachnoid hemorrhage)
B. Meningitis
C. Ischemic stroke
D. Encephalitis
E. Cerebral vascular embolism
Related question
A newborn with gestational age of 31 weeks presents with
hypotonia and depressed consciousness. Hematocrit is 35%,
general cerebrospinal fluid analysis shows increased content of
erythrocytes and protein, and low glucose. These data
correspond with the clinical presentation of:
A. Intracranial hemorrhage
B. Meningitis
C. Sepsis
D. Anemia
E. Intrauterine infection
37. A woman undergoing in-patient treatment for viral hepatitis
Objective
type B developed headache, nausea, recurrent vomiting,
Liver failure
memory lapses, flapping tremor of her hands, and rapid pulse. The patient is infected with viral hepatitis which is
Sweet smell from her mouth is detected. Body temperature is is a typical cause of acute liver failure.
37.6 C, heart rate is 89/min. What complication developed in
o

the patient?
A. Acute liver failure Acute liver failure
• Acute liver failure is a serious and potentially life-
B. Ischemic stroke
threatening condition that occurs when the liver suddenly
C. Gastrointestinal hemorrhage
loses its ability to function properly.
D. Hypoglycemic shock
• When the liver fails, it can detoxify blood from ammonia
E. Meningoencephalitis toxins.
• Ammonia toxins will accumulate in the brain leading to
these symptoms.
• Headache, nausea, recurrent vomiting, memory lapses,
flapping tremor of her hands.
• Fetor hepaticus: sweet, musty, or moldy odor caused by
accumulation of ammonia in blood.
• Causes:
• Viral hepatitis.
• Toxins: ex, mushroom toxins
• Medication
• Ischemic liver injury.
Acute liver failure
• Acute liver failure is a serious and potentially life-
2 hours after eating unknown mushrooms, a 28-year-old threatening condition that occurs when the liver suddenly
man sensed a decrease in his mobility and deterioration of loses its ability to function properly.
his ability to focus. This condition was then followed by a • When the liver fails, it can detoxify blood from ammonia
state of agitation and agression. On examiantion he is toxins.
• Ammonia toxins will accumulate in the brain leading to
disoriented and his speech is illegible. 4 hours later he
these symptoms.
developed fetor hepaticus and lost his consciousness. What
• Headache, nausea, recurrent vomiting, memory lapses,
syndrome can be observed in this patient?
flapping tremor of her hands.
A. Acute hepatic failure • Fetor hepaticus: sweet, musty, or moldy odor caused by
B. Hepatolienal syndrome accumulation of ammonia in blood.
C. Portal hypertension • Causes:
D. Cholestatic syndrome • Viral hepatitis.
E. Cytolytic syndrome • Toxins: ex, mushroom toxins
• Medication
• Ischemic liver injury.
38. A 43-year-old man, a coal-face worker with 15-year-long
record of work, complains of cough, thoracic pain, and dyspnea. Objective
The cough is mild, usually dry, occurs mostly in the morning. • Carboconiosis, also known as coal workers'
pneumoconiosis or black lung disease, is a
The pain is localized in the interscapular region and aggravates
lung disease that results from the inhalation of
during a deep intake of breath. Dyspnea occurs during physical
coal dust over a prolonged period of time.
exertion. Vesicular respiration in the lungs is weakened. Heart
• The coal dust particles can build up in the
sounds are rhythmic, heart rate is 86/min., blood pressure is lungs and cause inflammation and scarring
135/80 mm Hg. The abdomen is soft and painless. X-ray shows (fibrosis).
micronodular pulmonary fibrosis. Make the provisional
diagnosis:
A. Carboconiosis
B. Byssinosis
C. Siderosis
D. Berylliosis
E. Metal pneumoconiosis
39. Having examined a 52-year-old patient, the doctor Objective
diagnosed him with obesity (body mass index - 34 kg/m2, waist • Metabolic syndrome is a cluster of conditions
circumference - 112 cm) and arterial hypertension (170/105 mm that occur together, increasing the risk of
Hg). 2-hour postprandial blood sugar is 10.8 mmol/L. What developing cardiovascular disease, stroke, and
biochemical blood analysis needs to be conducted to diagnose type 2 diabetes.
• Lipid profile is essential for the diagnosis of
the patient with metabolic syndrome X?
metabolic syndrome.
A. Lipid profile
• Diagnosis is made when at least 3 of the
B. Bilirubin
following conditions are present:
C. Calcium and phosphorus 1. Abdominal obesity: A waist circumference of more
D. Creatinine and urea than 102 cm (40 inches) in men and more than 88 cm
E. Electrolytes (35 inches) in women.
2. High blood pressure: A blood pressure of 130/85
mmHg or higher, or the use of blood pressure
medication.
3. High blood sugar: A fasting blood glucose level of
100 mg/dL or higher, or the use of diabetes
medication.
4. High triglycerides: A blood triglyceride level of 150
mg/dL or higher, or the use of lipid-lowering
medication.
5. Low HDL cholesterol: An HDL cholesterol level of
less than 40 mg/dL in men and less than 50 mg/dL in
women.
40. After overexposure to cold a 45year-old woman developed
Objective
acute pain in her suprapubic and lumbar areas during urination,
Presence of leukocytes and hematuria typical for
sharp pains at the end of urination, false urges to urinate. Urine urinary tract infection.
is turbid with blood streaks. The doctor suspects urinary tract
infection. What results of laboratory analysis would be the most
indicative of such infection?
A. Leukocyturia, gross hematuria
B. Gross hematuria Urinary tract infection
C. Increased blood creatinine and blood urea • Cystitis
D. Daily proteinuria under 3.0 • Infection of bladder – Lower urinary tract
E. Daily proteinuria over 3.0 • Symptoms: dysuria (pain with urination), frequency
(going a lot), urgency (always feel like you must go), and
suprapubic pain.
• Pyelonephritis
• Infection of kidney – Upper urinary tract
• Symptoms: systemic symptoms (fever, chills), flank pain,
and CVA tenderness.
• Most infections ascend: Urethra → Cystitis → Pyelonephritis
• Diagnosis:
• Urinalysis: cloudy urine, leukocyte esterase (produced by
WBCs in urine) • Nitrites >10WBC/hp
• Culture: bacteriological inoculation >100,000 CFUs
Related questions
Find the keywords and answer

A 9-year-old girl complains of fever up to 37,5oC, A woman is on the 32nd week of her second pregnancy. She
headache, inertness, weakness, loss of appetite, complains of fever, chills, nausea, vomiting, lumbar pain, and
stomachache, and frequent painful urination. dysuria. Costovertebral angle tenderness is present on both
Provisional diagnosis of acute pyelonephritis is sides. Urine analysis: pyuria bacteriuria. Blood test:
made. Clinical urine analysis: specific gravity - 1018, leukocytosis. What is the most likely diagnosis?
no protein, leukocytes - 10-15 in the vision field. A. Cystitis
What investigation method can verify the diagnosis of B. Pyelitis
urinary C. Gestational pyelonephritis
system infection? D. Glomerulonephritis
A. Bacteriological inoculation of urine E. Latent bacteriuria
B. Rehberg test (creatinine clearance test)
C. Zymnytsky test (measurement of daily diuresis)
D. Complete blood count
E. Clinical urine analyses, dynamic testing
41. A 62-year-old woman was brought into the admission room
with complaints of severe burning retrosternal pain and Objective
asphyxia. She has a 10-year-long history of essential This patient presents with hypertensive crisis and
ST segment elevation.
hypertension. Objectively her condition is moderately severe.
• ST segment elevation is a typical finding in
She presents with skin pallor, cyanotic lips, and vesicular
myocardial infarction.
respiration over her lungs. The II heart sound is accentuated
• Myocardial infarction is a common
over the aorta. Blood pressure - 210/120 mm Hg, heart rate complication in hypertensive crisis patients.
(pulse) - 76/min. ECG shows elevation of ST segment in the
leads I, AVL, and V5-V6. What is the most likely diagnosis?
A. Hypertensive crisis complicated with acute myocardial
infarction
B. Uncomplicated hypertensive crisis
C. Hypertensive crisis complicated with instable angina
pectoris
D. Hypertensive crisis complicated with acute left ventricular
failure
E. Pulmonary embolism
42. A 35-year-old patient developed an epileptic attack with
tonoclonic spasms that lasted for 3 minutes. After the attack the
patient fell asleep but in 5 minutes the second attack occurred.
The first step of emergency aid would be to:
A. Ensure patency of airways
B. Take blood from the vein for analysis
C. Introduce diazepam intravenously
D. Prescribe antiepileptic drugs
E. Administer chloral hydrate via an enema
Objective
• Airway: Open the person's airway if it's
blocked.
• Breathing: Check for breathing and
provide oxygen if needed.
• Circulation: Check for a pulse and
control bleeding.
• Disability: Assess the person's
neurological function.
• Exposure: Remove clothing to fully
assess the person's injuries.
Objective
43. A 27-year-old woman, a teacher in the elementary school,
Irritable bowel syndrome – ‫اﻟﻘوﻟون اﻟﻌﺻﺑﻲ‬
complains of frequent stools, up to 3 times per day, with lumpy
• Irritable bowel syndrome is a recurrent
feces and large amount of mucus, abdominal pain that gradually abdominal pain with altered bowel habit
abates after a defecation, irritability. Her skin is pale and icteric. without unique organic pathology.
Pulse is 74/min., rhythmic, can be characterized as satisfactory. • Symptoms:
Blood pressure is 115/70 mm Hg. The abdomen is soft, 1. Abdominal pain or discomfort
moderately tender along the colon on palpation. Fiberoptic 2. Gas and bloating.
colonoscopy detects no changes. What disease can be 3. Mucus in the stool.
suspected? 4. Changes in bowel habits: diarrhea,
A. Irritable bowel syndrome constipation, or alternating bouts of both
B. Chronic non-ulcerative colitis • Aِِssociated with mood disorders (anxiety,
C. Chronic enteritis depression).
D. Crohn disease (regional enteritis)
E. Whipple disease
44. A 72-year-old man complains of lower extremity edema,
sensation of heaviness in the right subcostal area, dyspnea at
Objective
• Patient has been suffering from COPD in for
rest. For over 25 years he has been suffering from COPD.
25 years →
Objectively: orthopnea, jugular venous distention, diffuse • Increased blood pressure in the pulmonary
cyanosis, acrocyanosis. Barrel chest is observed, on percussion artery →
there is a vesiculotympanitic (bandbox) resonance, sharply • right heart can’t pump blood to the pulmonary
weakened vesicular respiration on both sides, moist crepitant artery →
crackles in the lower segments of the lungs. Heart sounds are • Cor pulmonale / chronic pulmonary heart: right
weakened, the II heart sound is accentuated over the pulmonary heart failure.
artery. The liver is +3 cm. What complicated the clinical course
of COPD in this patient?
A. Chronic pulmonary heart
B. Pulmonary embolism
C. Acute left ventricular failure
D. Diffuse pneumosclerosis
E. Community-acquired pneumonia
45. A 72-year-old man with pneumonia complains of marked Objective
dyspnea, chest pain, severe cough with expectoration, to is 39.5- Shock ‫ھﺑوط ﺣﺎد ﻓﻲ اﻟدورة اﻟدﻣوﯾﺔ‬
40oC, no urination for a whole day. Objectively the patient is Blood pressure of systolic less than 90 😳 is a sign
conscious. Respiratory rate is 36/min. Over the right lower of shock and requires immediate supportive
pulmonary lobe percussion sound is dull; on auscultation there treatment. In addition, he has anuria and marked
is bronchial respiration and numerous moist crackles. Blood dyspnea.
pressure is 80/60 mm Hg. Heart rate is 120/min. Heart sounds
are muffled, there is tachycardia. What tactics should the family
doctor choose in the management of this patient?
A. Hospitalization into the intensive care unit
B. Outpatient treatment
C. Treatment in the day patient facility
D. Hospitalization into the pulmonology unit
E. Hospitalization into the neurology unit
46. 2 hours after eating unknown mushrooms, a 28-year-old
man sensed a decrease in his mobility and deterioration of his
Objective
ability to focus. This condition was then followed by a state of
Amanita phalloides (death cap mushroom) is a
agitation and agression. On examiantion he is disoriented and mushroom toxin that can lead to acute liver toxins
his speech is illegible. 4 hours later he developed fetor hepaticus
and lost his consciousness. What syndrome can be observed in
this patient? Acute liver failure
A. Acute hepatic failure • Acute liver failure is a serious and potentially life-
B. Hepatolienal syndrome threatening condition that occurs when the liver suddenly
C. Portal hypertension loses its ability to function properly.
D. Cholestatic syndrome • When the liver fails, it can detoxify blood from ammonia
E. Cytolytic syndrome toxins.
• Ammonia toxins will accumulate in the brain leading to
these symptoms.
• Headache, nausea, recurrent vomiting, memory lapses,
flapping tremor of her hands.
• Fetor hepaticus: sweet, musty, or moldy odor caused by
accumulation of ammonia in blood.
• Causes:
• Viral hepatitis.
• Toxins: ex, mushroom toxins
• Medication
• Ischemic liver injury.
47. A 36-year-old man complains of marked dyspnea and
Objective
cardiac pain. He ascribes his disease to the case of influenza that • The findings of this patient is typical for
he had 2 weeks ago. Objectively he leans forward when sitting. Pericardial effusion which is treated with
The face is swollen, cyanotic, cervical veins are distended. Pericardial puncture (pericardiocenthesis).
Heart borders are extended on the both sides, heart sounds are • Pericardial puncture: accumulation of fluid in
muffled, heart rate = Ps = 118/min., BP is 90/60 mm Hg. Blood the pericardial sac between visceral
test: ESR is 46 mm/hour. ECG shows low voltage. Xray shows pericardium and the parietal pericardium.
trapezoidal cardiac silhouette and signs of pulmonary
congestion. Choose the treatment tactics:
A. Pericardial puncture (pericardiocenthesis)
B. Diuretics
C. Antibiotics
D. Pericardectomy
E. Glucocorticosteroids
48. A 39-year-old man suffers from chronic rheumatic heart
Objective
disease. He complains of dyspnea during physical exertion,
• Rheumatic heart disease (RHD) is a condition
cough with expectoration, and palpitations. Ausculation detects
that can develop after an individual has had an
intensified I heart sound and diastolic murmur; the sound of episode of rheumatic fever, which is a bacterial
opening mitral valve can be auscultated at the cardiac apex. The infection caused by group A streptococcus.
II heart sound is accentuated over the pulmonary artery. The • In rheumatic heart disease, the most effect
patient is cyanotic. X-ray shows dilated pulmonary root and valve 90% is mitral valve. However, it can
enlargement of the right ventricle and left atrium. What is the effect any of the four valve.
most likely diagnosis?
A. Mitral stenosis
B. Aortic stenosis
C. Pulmonary artery stenosis
D. Coarctation of the aorta
E. Patent ductus arteriosus
Valve auscultation points
1. Pulmonary (Semilunar) valve.
• 2 – left
2. Aortic (Semilunar) valve
• 2 – right
3. Tricuspid valve:
• Xiphoid process / 5th (right)
4. Mitral (bicuspid) valve:
• Heart apex – 5th midclavicular (Left)

Order of sounds
1. First sound of the heart S1:
• Closure of atrioventricular valves:
tricuspid and mitral
2. Second sound of the heart S2 :
• Closure of pulmonary and aortic.
49. A 23-year-old man complains of severe pain in his left knee
joint. Objectively the left knee joint is enlarged, with hyperemic Objective
skin, painful on palpation. Complete blood count: erythrocytes - • Joint pain and ↑ PT is a typical findings in
patients with hemophilia
3.8 · 1012/L, Hb- 122 g/L, leukocytes - 7.4 · 109/L, platelets 183
· 109/L. Erythrocyte sedimentation rate - 10 mm/hour. Bleeding
time (Duke method) - 4 min., Lee-White coagulation time - 24 Bleeding disorders
Types of bleeding disorders
min. Partial thromboplastin time (activated) - 89 seconds.
• Coagulation factors disorders:
Rheumatoid factor - negative. What is the most likely
• Deep bleeding: Joint bleeding, deep tissue
diagnosis?
bleeding
A. Hemophilia, hemarthrosis • Disorders:
B. Werlhof disease (immune thrombocytopenia) 1. Hemophilia A, B, or C: ↑ PT
C. Rheumatoid arthritis 2. Vitamin K deficiency
D. Thrombocytopathy • Platelets disorders:
E. Hemorrhagic vasculitis (Henoch-Schonlein purpura), • Superficial bleeding: Mucosal bleeding,
articular form skin bleeding, petechiae
• Disorders: Bernard-Soulier syndrome,
Glanzmann thrombasthenia, Immune
thrombocytopenia, Uremic platelet
dysfunction
• Mixed: von Willebrand disease and
Disseminated intravascular coagulation
Bleeding disorders Bleeding
Types of bleeding disorders Types of bleeding disorders
• Coagulation disorders: 1. Partial Thromboplastin Time (PTT ): measure
• Deep bleeding: Joint bleeding, deep tissue function of intrinsic factors pathway
bleeding 2. Prothrombin Time (PT): measure function of
• Disorders: extrinsic factor pathway
1. Hemophilia A, B, or C: ↑ PT 3. Bleeding Time:
2. Vitamin K deficiency • Time it takes for bleeding to stop after a
• Platelets disorders: standardized skin puncture
• Superficial bleeding: Mucosal bleeding, • Test of platelet function.
skin bleeding, petechiae • 2-9 minutes.
• Disorders: Bernard-Soulier syndrome, 1. Lee-White coagulation time (old):
Glanzmann thrombasthenia, Immune • Time it takes for blood to clot.
thrombocytopenia, Uremic platelet • 8-15 minutes
dysfunction
• Mixed
50. A 24-year-old woman, a kindergarten teacher, has been sick
for 2 days already. Disease onset was acute. She presents with
elevated body temperature up to 38.0oC, pain attacks in her
lower left abdomen, liquid stool in small amounts with blood
and mucus admixtures 10 times a day. Pulse - 98/min., blood
pressure - 110/70 mm Hg. Her tongue is moist and coated with
white deposits. The abdomen is soft, the sigmoid colon is
painful and spastic. Make the provisional diagnosis:
A. Shigellosis
B. Escherichiosis
C. Salmonellosis
D. Yersiniosis
E. Rotavirus infection

Objective
Bloody diarrhea and sigmoid colon (large
intestine) spasm is typical for shigella infection.
Thyroxin / triiodothyronine T3 T4 🏃 🔥
51. A 38-year-old woman complains of weakness, sleepiness,
pain in the joints, weight gain despite low appetite, and 1. From: follicular cells of thyroid gland
constipations. She presents with dry and thickened skin, puffy 2. Contains: iodine
3. Function: regulate Basal metabolic rate. (‫)اﻟﺣرق‬
and amimic face, narrowed palpebral fissures, thick tongue, and
4. How? By process of disjunction of oxidation and
deep hoarse voice. Her heart sounds are weak, pulse is 56/min. oxidative phosphorylation
Low levels of free T4 are observed. This patient needs to take 5. Hyperthyroidism:
the following on a regular basis: • Increase basal metabolic rate (‫= )اﻟﺣرق‬
A. Thyroxine 1. Loss of weight.
2. Hyperthermia.
B. Mercazolil (Thiamazole)
3. Tachycardia
C. Lithium carbonate 4. Sweating.
D. Furosemide 5. Tremor ‫اﻟرﺟﻔﺔ‬
E. Calcium gluconate • Exophthalmia ‫ﺟﺣوظ اﻟﻌﯾن‬: by increasing
Glycosaminoglycans → water accumulation in the
eye.
6. Hyp0rthyroidism:
• Decrease basal metabolic rate (‫= )اﻟﺣرق‬
1. Weight gain.
2. Growth retardation.
3. Disproportional body build.
4. Bradycardia.
5. Physical and mental retardation.
7. Treatment: replacement therapy .
Thyroxin / triiodothyronine T3 T4 🏃 🔥

1. From: follicular cells of thyroid gland


2. Contains: iodine
3. Function: regulate Basal metabolic rate. (‫)اﻟﺣرق‬
4. How? By process of disjunction of oxidation and
oxidative phosphorylation
5. Hyperthyroidism:
• Increase basal metabolic rate (‫= )اﻟﺣرق‬
1. Loss of weight.
2. Hyperthermia.
3. Tachycardia
4. Sweating.
5. Tremor ‫اﻟرﺟﻔﺔ‬
• Exophthalmia ‫ﺟﺣوظ اﻟﻌﯾن‬: by increasing
Glycosaminoglycans → water accumulation in the
eye.
6. Hyp0rthyroidism:
• Decrease basal metabolic rate (‫= )اﻟﺣرق‬
1. Weight gain.
2. Growth retardation.
3. Disproportional body build.
4. Bradycardia.
5. Physical and mental retardation.
7. Treatment: replacement therapy .
52. A 23-year-old man has accidentally swallowed brake fluid.
After that he has been presenting with anuria for 5 days already; Signs of kidney damage (↓ GFR )
his creatinine levels elevated up to 0.569 mmol/L. What 1. ↑ Creatinine: > 0.106 μmol/L (106 mmol/L).
treatment tactics should be chosen in this case? 2. Hyperkalaemia > 5mmol/L
A. Detoxication therapy 3. High BUN: > 15
B. Antidotal therapy
C. Hemodialysis
D. Diuretics
Indications for dialysis
E. Plasmapheresis 1. Acute or chronic kidney injury: ↓ GFR
(Anuria) = ↑ toxins (↑ creatinine indicator).
2. Drug overdose or toxicity: ethylene glycol
(break fluid)
3. Fluid overload: Dialysis may be necessary to
remove excess fluid.
4. Electrolyte imbalances: Dialysis may be
necessary to correct electrolyte imbalances,
such as high potassium levels, that can occur
when the kidneys are not functioning
properly.
53. A 52-year-old man for the last 3 years has been suffering Objective
from difficult swallowing of solid food, burning retrosternal Esophageal carcinoma
pain that aggravated during eating, loss of body mass, and • Squamous cell or adenocarcinoma
occasional vomiting with undigested food. Esophageal X-ray • Both types: ↑ risk in smokers
shows S-shaped deformation of the esophagus and its dilation; • Presents with narrowing of the esophagus
at the cardiac orifice the esophagus is constricted; esophageal (tumor growth) and progressive dysphagia
mucosa is smooth, without signs of peristalsis. Make the • Starts with solids
provisional diagnosis: • Progresses to liquids as tumor grows
A. Esophageal carcinoma • Other symptoms:
B. Diaphragmatic hernia • Weight loss
C. Esophageal achalasia • Hematemesis
D. Reflux esophagitis
E. Esophageal diverticulum
Objective
Related question
Esophageal Achalasia
A 38-year-old man is complaining of periodical problematic
• What is it? failure of LES to relax due to
swallowing of both solid and liquid foods that is observed for
degeneration of inhibitory neurons in the
many months. Sometimes he develops an intense retrosternal
myenteric (Auerbach) plexus of esophageal
pain, especially after hot beverages. Asphyxia attacks are
wall.
observed at night. He has no weight loss. His general condition
• Presents with: progressive dysphagia to solids
is satisfactory, the skin is of normal color. Examination detects
and liquids (vs obstruction—primarily solids),
no changes in the gastrointestinal tract. Chest X-ray shows
retrosternal pain.
dilation of the esophagus with air-fluid levels in it. Make
• Associated with: ↑ risk of esophageal cancer.
the right diagnosis:
• Manometry findings include uncoordinated or
A. Esophageal achalasia
absent peristalsis with ↑ LES resting pressure.
B. Esophageal cancer
• Barium swallow shows dilated esophagus with
C. Gastroesophageal reflux disease
area of distal stenosis (“bird’s beak” A ).
D. Myasthenia
• Treatment: surgery, endoscopic procedures
E. Esophageal candidiasis
(eg, botulinum toxin injection).
Heart X-ray of a 31-year-old man has revealed the
following: with tightly filled opacified esophagus there Esophageal carcinoma
is a marginal filling defect in its middle third on the • Squamous cell or adenocarcinoma
posterior wall; the defect is 1.8x1.3 cm in size with clear • Both types: ↑ risk in smokers
oval border. Mucosal folds are retained and envelop the • Presents with narrowing of the esophagus
defect; wall peristalsis and elasticity are not affected. (tumor growth) and progressive dysphagia
There are no complaints regarding the condition of the • Starts with solids
patient’s alimentary canal. Make the provisional • Progresses to liquids as tumor grows
diagnosis: • Other symptoms:
A. Esophageal tumor • Weight loss
B. Achalasia cardiae • Hematemesis
C. Esophageal burns
D. Diverticulum
E. Barrett esophagus
A47-year-old female patient complains of having pain on
swallowing and difficult passing of solid food for two months.
The patient has taken to the liquid and semi-liquid food. During
the last week the liquid food has barely passed through. General
condition is satisfactory, the patient is undernourished, the
appetite is preserved, there is a fear of eating. What is the Esophageal carcinoma
provisional diagnosis? • Squamous cell or adenocarcinoma
A. Esophageal carcinoma (memorise!) • Both types: ↑ risk in smokers
B. Esophageal stricture • Presents with narrowing of the esophagus
C. Esophageal foreign body (tumor growth) and progressive dysphagia
D. Esophageal varices • Starts with solids
E. Esophageal achalasia • Progresses to liquids as tumor grows
• Other symptoms:
A 45-year-old man was delivered to a hospital with complaints • Weight loss
of vomiting with streaks of blood, loss of weight. On • Hematemesis
esophagofiberscopy a cauliflower-shaped mucosal growth was
detected in the abdominal esophagus. The mucosa there bleeds
on contact. What preliminary diagnosis can be made?
A. Esophageal tumor
B. Barrett esophagus
C. Abdominal esophagitis
D. Esophageal diverticulum
E. Esophageal achalasia
54. A 53-year-old man complains of general weakness, loss of Objective
appetite, and painful vesicles appearing on his skin. The disease Acantholytic pemphigus
onset occurred suddenly, after hyperinsolation one week ago. • Terminology:
Examination detects isolated vesicles with wrinkled opercula • Pemphigoid: blister or bubble
and occasional painful erosions on the skin of the patient’s torso • Acantholysis: separation of skin
and limbs. Nikolsky sign is positive. What is the most likely • What is it? Autoimmune disease in which
diagnosis? immune system produces antibodies that attack
proteins in the skin and mucous membranes,
A. Acantholytic pemphigus
leading to the formation of blisters and
B. Nonacantholytic pemphigus
erosions..
C. Duhring’s disease (dermatitis herpetiformis)
D. Herpes
E. Toxicodermia
Related Pemphigus neonatorum
A 3-week-old infant developed large, flaccid vesicles with Pemphigus neonatorum is a rare autoimmune
purulent contents on the skin of chest and abdomen. The disease caused by the transfer of maternal
vesicles rupture quickly. Make the provisional diagnosis: antibodies against a specific protein, called
A. Pemphigus neonatorum desmoglein 1, to the baby during pregnancy. These
B. Vesiculopustulosis antibodies attack the baby's skin and mucous
C. Toxic erythema membranes, causing blistering and erosions.
D. Pemphigus syphiliticus Blisters can get infected resulting in formation of
E. Pseudofurunculosis purulent inflammation.
55. A patient is being treated in the tuberculosis clinic. Objective
Throughout the last 3 weeks he has been suffering from Meningitis symptoms (headache & nuchal
headaches of increasing intensity. Neurological examination rigidity) + Tuberculosis infection → Tuberculous
detects nuchal rigidity without focal signs. Make the provisional meningitis
diagnosis:
A. Tuberculous meningitis
B. Chorea minor
C. Brain tumor
D. Myelitis
E. Convexital arachnoiditis
56. A patient has gradually lost consciousness. The skin is pale Objective
and dry. There is a smell of ammonia from the mouth. ↑ Urea + lost consciousness = Uremic coma
Respirations are deep and noisy. Heart sounds are muffled,
pericardial friction rub is present. Blood pressure is 180/130
mm Hg. Blood test: Нb- 80 g/L, leukocytes 12 · 109/L, blood
glucose - 6.4 mmol/L, urea 50 mmol/L, creatinine - 1200 Uremia—syndrome resulting from high
mcmol/L, blood osmolarity - 350 mOsmol/L. No urinary serum urea.
excretion. Make the diagnosis:
A. Uremic coma Can present with:
B. Hyperglycemic coma 1. Pericarditis
C. Acute renal failure 2. Encephalopathy (unconsciousness)
D. Acute disturbance of cerebral circulation 3. Anorexia
4. Nausea
E. Hyperosmolar coma
(Ure- PEAN 🥜)
57. A 72-year-old man diagnosed with ischemic heart
disease presents with diffuse cardiosclerosis, permanent Objective
• This patient presents with symptoms of
tachysystolic atrial fibrillation, heart failure IIа, FC III.
chronic heart failure which can be treated
Objective examination of vital signs: blood pressure is
with digoxin.
135/80 mm Hg, heart rate is 160/min., pulse is 125/min.
• EF (SV/EDV) is an index of ventricular
Left ventricular ejection fraction is 32%. What drug is
contractility (↓ in systolic HF; usually
indicated in this case and should be presribed to the normal in diastolic HF). Normal is 45%
patient?
A. Digoxin
B. Procainamide (Novocainamide)
C. Isadrine (Isoprenaline) Acute heart failure drugs
D. Verapamil • Glycoside:
E. Ivabradine • Corglycon.
• Strophanthin
• Non-glycoside
• Dobutamine
Chronic heart failure drugs
• Digoxin (Glycoside):
• Uses: Chronic heart failure & Anti-arrhythmia.
• Note: hypokalemia or hypercalcemia will cause
digoxin toxicity.
• Antidotes: Unithiol (sulfa)
Left ventricular heart failure
Causes:
• Increase in Afterload (hypertention).
Related • Mitral stenosis
Symptoms:
A 13-year-old girl for the last two weeks has been complaining
• No blood pumping to the body →
of dyspnea and shin and foot edemas that appear after a physical
• Blood accumulate →
exertion. In the morning the edemas significantly decrease. • Edema, dyspnea and other symptoms.
Clinical examination revealed enlarged liver and coarse systolic
murmur over the heart area. Blood test and urinalysis are
without changes. What is the most likely cause of edemas in this
child?
A. Heart failure
B. Nephrotic syndrome
C. Acute pyelonephritis
D. Angioneurotic edema
E. Hepatic cirrhosis
58. A 34-year-old man complains of pale edema of the face, Objective
feet, shins, and lumbar area, elevated blood pressure up to The involvement of systemic organs in this
160/100 mm Hg, and general weakness. He has a clinical patient, particularly the kidneys, indicates that the
history of nonspecific ulcerative colitis. Objectively: pulse - patient may be suffering from systemic
84/min., rhythmic, blood pressure - 165/100 mm Hg; edemas all amyloidosis
over the body; the skin is pale and dry, with low turgor. The
Amyloidosis
kidneys cannot be palpated, on an attempt to palpate them they
Accumulation of amyloid protein
are painless. Blood test: erythrocytes - 3.0·1012/L, Нb- 100 g/L, (misfolded proteins) in different
erythrocyte sedimentation rate - 50 mm/hour. Urinalysis: organs. It can be systemic or localised.
proteins 3.5 g/L, erythrocytes - 7-10 in the vision field, Diagnosis can be made by finding
leukocytes - 5-6 in the vision field. Daily proteinuria - 6 grams. amyloid depositions in the gingiva.
What analysis should be conducted additionally to verify the • Kidney: most commonly involved
diagnosis? organ → nephrotic syndrome
A. Gingival biopsy for the diagnosis of amyloid disease (proteinuria) and hypertension
B. Radioisotopic examination of kidneys • Heart → heart failure
C. Urinalysis for Bence-Jones protein • Liver → liver failure/
D. Renal ultrasound
E. Survey and excretory urography
59. A 42-year-old man, a worker at the meat processing factory, Objective
developed an itching spot on his lower jaw, which gradually The antibiotics commonly used to treat anthrax
transformed into a slightly painful carbuncle 3 cm in diameter, are ciprofloxacin, doxycycline, and penicillin.
surrounded by a painless swelling that reaches the clavicle.
Temperature is subfebrile, under 37.8oC. The doctor suspects
anthrax. What drug should this man be prescribed for treatment?
A. Penicillin
B. Levomycetin (Chloramphenicol)
C. Biseptol (Co-trimoxazole)
D. Interferon alpha
E. Azidothymidin (Zidovudine)
60. A 57-year-old patient complains of dyspnea at rest. The Objective
patient presents with orthopnea, acrocyanosis, bulging cervical The patient suffers from pleural effusion is
veins. On percussion: dull sound over the lower lung segments. accumulation of fluid in the pleural cavity.
On auscultation: no respiratory sounds. Heart rate is 92/min. • Types of pleural effusion:
Right-sided cardiac dilatation is observed. The liver is +7 cm. • Exudate: fluid is reach in cells and
Shins are swollen. Pleural effusion is suspected. What indicator proteins ( > 25g/l)
would confirm the presence of transudate in this case? • Transudate: fluid is clear (protein <25 g/l)
A. Total protein content in the pleural fluid below 25 g/L
B. Presence of atypical cells
C. Total protein content in the pleural fluid exceeding 30 g/L
D. Specific gravity exceeding 1015
E. Positive Rivalta’s test
61. A 33-year-old man developed multiple rashes on the skin of Objective
his torso and extensor surfaces of his upper and lower limbs. • Psoriasis is a chronic autoimmune condition
The rashes itch and occasionally fuse together and form that affects the skin causing papules and
plaques. The elements of rash are covered with silver-white fine plaques with silvery scaling.
scales that easily flake off when scratched. Grattage test results • Positive Grattage test (Auspitz sign): pinpoint
in three sequential phenomena: stearin spot, terminal film, and bleeding when scaled off.
punctate hemorrhage. What diagnosis can be suspected?
A. Psoriasis
B. Parapsoriasis
C. Pyoderma
D. Lichen ruber planus
E. Secondary papular syphilid
62. A 38-year-old woman after physical overexertion suddenly Objective
developed palpitations, dyspnea, and a dull pain in the cardiac Small irregular P wave+ irricular R-R interval is a
area. For 10 years she has been registered for regular check-ups key feature in atrial fibrillation.
due to rheumatism and mitral valve disease with non-disturbed
blood circulation. Oblectively her pulse is 96/min., of unequal
strength. Blood pressure is 110/70 mm Hg, heart rate is
120/min. ECG registers small unevenly-sized waves in place of
P-waves, R-R intervals are of unequal length. What is the most
likely diagnosis?
A. Atrial fibrillation
B. Paroxysmal supraventricular tachycardia
C. Atrial flutter
D. Paroxysmal ventricular tachycardia
E. Respiratory arrhythmia
63. An 18-year-old patient always obeys others and adapts his Objective
needs to the demands of the people on whom he depends. He The patient suffers from dependent personality
excessively defers to their wishes and makes them responsible disorder (DPD) is a mental health condition
for his wellbeing, cannot defend his interests and needs support characterized by an excessive need for
from other people. Such psychic profile has been formed in the reassurance, support, and guidance from others.
childhood, remains unchanged, and hinders adaptation. What “always obeys others”.
psychic disorder is observed in this patient?
A. Dependent personality disorder
B. Anxiety (avoidant) personality disorder
C. Anankastic personality disorder
D. Markedly accentuated personality
E. Psychopathy-like state
64. A 45-year-old man with thrombophlebitis of the deep veins Thromboembolism
in his legs suddenly after physical exertion developed sharp pain Question sequence
in his thorax on the right, dyspnea, and hemoptysis. Objectively Why do we fear from Deep vein Thrombosis (DVT) 🤔 ?
his condition is severe; he presents with acrocyanosis, 1. Thrmobosis in the lower limbs can become mobile →
shortening of pulmonary percussion sound on the right, and 2. Move throught the circulation →
3. Reach the the lungs (Pulmonary arteries) →
weakened respiration. Respiration is 30/min., blood pressure is
4. ischemia in the lungs →
110/80 mm Hg. ECG shows sinus tachycardia, heart rate is
5. Lung stop working →
120/min., electrical axis of the heart deviates to the right, SI-QIII.
6. Death 💀
What is the most likely diagnosis?
A. Pulmonary embolism
B. Community-acquired right-sided pneumonia
C. Cancer of the right lung
D. Right-sided exudative pleurisy
E. Spontaneous pneumothorax
Related
A 65-year-old woman, who had been suffering from deep vein
thrombophlebitis of the lower leg, suddenly died when awaiting
her appointment with the doctor. Autopsy revealed loose friable
red masses with corrugated dull surface in the main pulmonary
artery and its bifurcation. What pathologic process was
discovered by the pathologist in the pulmonary artery?
A. Fat embolism
B. Foreign body embolism
C. Thrombosis
D. Tissue embolism
E. Thromboembolism

Autopsy of the body of a man, who died during an abdominal


surgery, revealed numerous thrombi in the veins of the lesser Thromboembolism
pelvis. Clinically, thromboembolic syndrome was detected. Question sequence
Where should the doctor search for the embolus? Why do we fear from Deep vein Thrombosis (DVT) 🤔 ?
A. Brain 1. Thrmobosis in the lower limbs can become mobile =
B. Pulmonary arteries 2. Move throught the circulation =
C. Portal vein 3. Reach the the lungs (Pulmonary arteries) =
D. Left ventricle of heart 4. ischemia in the lungs =
E. Veins of the lower extremities 5. Lung stop working =
6. Death 💀
65. A 48-year-old woman has been hospitalized due to
Objective
development of tachysystolic atrial fibrillation. She has lost 5 kg
Tachycardia and loss of weigh are typical findings
of body weight within 2 months. On palpation there is a node in
in patients with hyperthyroid disorders like toxic
the left lobe of the thyroid gland. What pathology resulted in the nodular goiter
development of this condition?
A. Toxic nodular goiter
B. Aterosclerotic cardiosclerosis Objective
C. Chronic thyroiditis Toxic nodular goiter
D. Nontoxic nodular goiter • Focal patches of hyperfunctioning follicular
E. Autoimmune thyroiditis cells distended with colloid working
independently of TSH.
• Due to TSH receptor mutations in 60% of
cases.
• ↑ release of T3 and T4. Hot nodules.
• ‫اﻟﺜﺎﯾﺮوﯾﺪ ﺷﻐﻠﮫ ﻣﻦ دﻣﺎﻏﮭﺎ‬
Thyroxin / triiodothyronine T3 T4 🏃 🔥

1. From: follicular cells of thyroid gland


2. Contains: iodine
3. Function: regulate Basal metabolic rate. (‫)اﻟﺣرق‬
4. How? By process of disjunction of oxidation and
oxidative phosphorylation
5. Hyperthyroidism:
• Increase basal metabolic rate (‫= )اﻟﺣرق‬
1. Loss of weight.
2. Hyperthermia.
3. Tachycardia
4. Sweating.
5. Tremor ‫اﻟرﺟﻔﺔ‬
• Exophthalmia ‫ﺟﺣوظ اﻟﻌﯾن‬: by increasing
Glycosaminoglycans → water accumulation in the
eye.
6. Hyp0rthyroidism:
• Decrease basal metabolic rate (‫= )اﻟﺣرق‬
1. Weight gain.
2. Growth retardation.
3. Disproportional body build.
4. Bradycardia.
5. Physical and mental retardation.
7. Treatment: replacement therapy .
66. A 48-year-old woman developed insomnia, depressive Objective
mood, anxiety, fears and suicidal thoughts after the death of her Patient presentations are typical for depression
husband that occurred one month ago. During her stay in the disorder which can be treated with antidepressints
hospital she speaks in a low voice, is depressed, anxious, avoids
sleeping, refuses to eat. What medications should be prescribed
in this case?
A. Antidepressants
B. Antipsychotics
C. Group B vitamins
D. Nootropics
E. Anticonvulsants
Objective
67. A 32-year-old woman complains of episodes of intense fear • Hysterical neurosis: a group of psychological
that occur without visible cause and last for 10-20 minutes; the disorders characterized by physical symptoms
episodes are characterized by rapid pulse, sweating, labored without an apparent physical cause.
breathing, and vertigo. Specify the likely diagnosis: • Hypochondriacal neurosis: excessive anxiety or fear
A. Panic disorder about having a serious illness, despite having no or
B. Paranoid syndrome only mild physical symptoms
C. Manic syndrome • Depressive neurosis: prolonged and persistent
D. Simple schizophrenia feelings of sadness, hopelessness, and a loss of
E. Claustrophobia interest in activities that were once pleasurable.
• Obsessive neurosis: persistent and unwanted
thoughts, impulses, or images (obsessions) that cause
significant distress and anxiety
• Paranoid personality disorder: type of personality
disorder characterized by a pervasive distrust and
suspicion of others, including their motives and
intentions.
• Panic disorder is a type of anxiety disorder
characterized by recurring and unexpected panic
attacks. Symptoms such as sweating, trembling,
shortness of breath, chest pain, and a feeling of
choking or suffocation.
68. A 39-year-old man suffers from chronic adrenal Objective
insufficiency and receives replacement glucocorticoid therapy • Adrenal insufficiency: inability of adrenal
(hydrocortisone - 15 mg/day). He is to undergo elective surgery glands to generate enough glucocorticoids
for calculous cholecystitis. What medication adjustment should (cortisol) +/− mineralocorticoids (aldosterone)
be made on the day of the surgery to prevent the development of which can be treated with replacement therapy.
acute adrenal insufficiency? • During stress, such surgery or infection →
A. Increase the dosage by 2-3 times • The body requirements for cortisol is
B. Cancel the drug for the day of the surgery increased.
• The dose of cortisol should be increased.
C. Add a mineralocorticoid
D. Add an antibiotic
E. Prescribe a large volume intravenous fluid infusion
69. After a long drive with the window open a man developed
facial asymmetry; he cannot close his right eye, his right
nasolabial fold is smoothed out, movements of expression are
absent on the right, there is a disturbance of gustatory sensation
in the tongue on the right. No other neurological pathologies
were detected. What disease can be provisionally diagnosed in
this patient?
A. Neuropathy of the facial nerve Facial palsy
B. Neuropathy of the trigeminal nerve (Neuropathy of the facial nerve)
C. Trigeminal ganglionitis • Usually develops after HSV reactivation.
D. Neuropathy of the oculomotor nerve • Treatment: glucocorticoids +/– acyclovir. Most
E. Ischemic stroke patients gradually recover function, but
aberrant regeneration can occur.
• Other causes of peripheral facial palsy include
Lyme disease, herpes zoster (Ramsay Hunt
syndrome), sarcoidosis, tumors (eg, parotid
gland), diabetes mellitus..
70. A 56-year-old woman was diagnosed with stage 2
Objective
hypertension of the 2nd degree. She belongs to the group of
Nonselective β blocker can help treating
moderate risk and has bronchial asthma. What group of drugs is hypertension by decreasing heart rate, however, it
CONTRAINDICATED to this patient? can lead to blocking of β receptors in the lung
A. β-blockers increasing asthma symptoms.
B. Angiotensin-converting enzyme inhibitors
C. Diuretics
D. Calcium antagonists
E. Imidazoline receptor antagonists
71. A 45-year-old woman is registered for regular check-ups due Objective
to Werlhof disease (immune thrombocytopenia). Complete Patient findings don’t require urgent intervention
blood count: Нb- 100 g/L, erythrocytes 2.8·1012/L, platelets - and should continue the follow up with the
90.0·109/L, leukocytes - 8.4 · 109/L, erythrocyte sedimentation hematologist.
rate 13 mm/hour. Examination detects a single small hematoma
on the anterior surface of the thigh, developed after the patient Bleeding disorders
Types of bleeding disorders
accidentally stumbled on a table. What treatment tactics should
• Coagulation factors disorders:
be chosen in this case?
• Deep bleeding: Joint bleeding, deep tissue
A. Continue the supervision by the hospital hematologist
bleeding
B. Urgent hospitalization into the hematology unit • Disorders:
C. Urgently start a hemostatic therapy, followed by a planned 1. Hemophilia A, B, or C: ↑ PT
hospitalization into the hematology unit 2. Vitamin K deficiency
D. Urgent hospitalization into the general care unit • Platelets disorders:
E. Administer thrombocytic mass, continue the treatment in the • Superficial bleeding: Mucosal bleeding,
hematology unit skin bleeding, petechiae
• Disorders: Bernard-Soulier syndrome,
Glanzmann thrombasthenia, Immune
thrombocytopenia, Uremic platelet
dysfunction
• Mixed: von Willebrand disease and
Disseminated intravascular coagulation
72. The dermatologist has an appointment with a 30- Arthropods (‫)ﺣﺷرات‬
year-old man that complains of severely itching rashes
Scorpians:
that especially disturb him at night. The rashes • 4 pairs of ambulatory legs
developed 2 weeks ago, after he had returned from a • Segmented abdomen ‫ﺑطن ﻣﻘﺳﻣﺔ‬
travel. Objectively on the lateral surfaces of his fingers,
hands, wrists, elbows, lower abdomen, genitals, and Karakurt spider / Steppe spider/ black widow:
thighs there are paired papulovesicles, single pustules, • Roundish black abdomen. ‫ﺑطن ﺳوداء‬
and scratch marks. What disease can be suspected? • Two rows of red spots on its dorsal side.
A. Scabies • Four pairs of jointed limbs.
B. Pyoderma Phthiriasis:
C. Dermatitis • Feeds on the blood.
D. Eczema • Location: hair near armpits armpit, beard, eyebrows.
E. Shingles Dog-louse:
• Encephalitis.
• Body louse (Pediculus humanus humanus):
• Features: ~ 3 mm and white
Scabies:
• Skin itching.
• Between fingers, and genital areas.
Wohlfahrtia magnifica:
• Infective form: Larvae.
• Disease: Myiasis
• Feature: tissue necrosis.
73. A 38-year-old woman developed a medical condition 7 days
Objective
after her return from Bangladesh. Periodical elevation of
Malaria patient & loss of consciousness →
temperature was accompanied by chills and excessive sweating.
Cerebral coma
She was diagnosed with tropical malaria. Next day her
condition further deteriorated: body temperature - 38oC,
inertness, periodical loss of consciousness, generalized seizures,
tachycardia, hypotension, and icteric skin. What complication
can be suspected in this case?
A. Cerebral coma
B. Serous meningitis
C. Purulent meningitis
D. Acute hepatic failure
E. Acute heart failure
74. A 73-year-old woman came to the family physician for one
Objective
of her regular follow-up examinations. Three months ago she
Metformin is first line drug for treating type 2
was found to have type 2 diabetes mellitus. She was keeping to diabetes mellitus.
her diet and exercise plan and taking phytopreparations. On
examination her fasting glucose was within the range of 7.8-8.6
mmol/L, HbА1с - 7.9%. Height - 164 cm, weight - 83 kg. What
blood sugar-controlling medicine should she be prescribed first
in the course of her pharmacological therapy?
A. Metformin
B. Glibenclamide
C. Glimepiride
D. Gliclazide
E. Insulin
75. A 27-year-old man complains of pain in his leg joints,
Objective
purulent discharge from the eyes, and painful burning sensations
during urination. Disease onset was acute. He has a history of
• Chlamydia is a sexually transmitted infection
influenza. The patient smokes and drinks alcohol in excess. In (STI) caused by the bacterium Chlamydia
his line of work he is often away on business trips. What is the trachomatis.
most likely etiological factor of this disease? • Male symptoms:
A. Chlamydia • Pain or burning sensation when urinating.
B. Adenovirus • Eye inflammation and discharge.
C. Streptococci • Discharge from the urethra, which may be
D. Staphylococci clear, white, or yellow in color.
E. Candida • Pain or swelling in the testicles.
• Female symptoms:
• Vaginal discharge, which may be clear,
white, or yellow in color
• Dysuria: Pain or burning sensation when
urinating
• Dyspareunia: Pain during sexual
intercourse
• Abdominal pain or pelvic pain
• Intermenstrual bleeding: Bleeding
between periods or after sex
76. A 46-year-old woman has diarrhea with abdominal
distension, loss of body mass, and large amounts of porridge- Objective
like foul smelling stool without blood streaks or tenesmus. • According to the patient's presentation, he has
Objective examination detects moderate tenderness in the indigestion of fats (steatorrhea) and proteins
(creatorrhea) which is typical for pancreatic
mesogastrium and left abdominal flank. Feces analysis detects
insufficiency.
steatorrhea with neutral fat and creatorrhea. What prescription
• Pancreatic insufficiency a condition in which
would be the most advisable in this case?
pancreas does not produce enough digestive
A. Multi-enzyme preparations enzymes to properly break down and absorb
B. Cholinergic antagonists nutrients from food.
C. Metronidazole and loperamide • Treatment for pancreatic insufficiency often
D. Antacids and antispasmodics involves enzyme replacement therapy (ERT),
E. Cholinergic antagonists and antibacterial agents which involves taking oral pancreatic enzyme
supplements with meals.
• ERT includes lipase, amylase, and protease
enzymes, which help break down fats,
carbohydrates, and proteins in the food
77. A man was brought into the admission room after an
overexposure to cold. He complains of sharp pain in the
small of his back and elevated body temperature up to
38oC. He took some aspirin. Blood test: leukocytes - 10.5
· 1012/L, eosinophils - 5%, band neutrophils - 8%,
segmented neutrophils - 51%, lymphocytes - 32%,
monocytes - 4%, erythrocyte sedimentation rate - 28
mm/hour. Urinalysis: protein - 0.6 g/L, leukocytes - cover Urinary tract infection
the whole vision field, large amount of mucus. What is • Cystitis
the most likely diagnosis? • Infection of bladder – Lower urinary tract
A. Acute pyelonephritis • Symptoms: dysuria (pain with urination), frequency
B. Chronic pyelonephritis (going a lot), urgency (always feel like you must go), and
suprapubic pain.
C. Acute glomerulonephritis
• Pyelonephritis
D. Tubulointerstitial nephritis
• Infection of kidney – Upper urinary tract
E. Subacute malignant glomerulonephritis
• Symptoms: systemic symptoms (fever, chills), flank pain,
and CVA tenderness.
• Most infections ascend: Urethra → Cystitis → Pyelonephritis
Note • Diagnosis:
Chronic pyelonephritis: happens due to recurrent • Urinalysis: cloudy urine, leukocyte esterase (produced by
episodes of pyelonephritis; fibrosis and long time WBCs in urine) • Nitrites >10WBC/hp
is a key. • Culture: bacteriological inoculation >100,000 CFUs
78. A 26-year-old man complains of chills, rhinitis, dry cough, Rubella ‫اﻟﺤﺼﺒﺔ اﻷﻟﻤﺎﻧﯿﺔ‬
and fever up to 38oC. Examination shows him to be in a • Rubella, also known as German measles, is a viral
moderately severe condition; there are small pale pink non- infection that is generally mild in children but can
merging spots on the skin of his back, abdomen, and be more serious in adults, especially in pregnant
extremities. Palpation reveals enlarged occipital and axillary women. The symptoms of rubella in adults may
lymph nodes. No information about vaccination history could be include:
obtained. What is the likely etiology of this disease? 1. Rash: A rash that begins on the face and then
A. Rubella virus spreads to the rest of the body is a common
symptom of rubella
B. Epstein-Barr virus
2. Fever: Adults with rubella may develop a
C. Streptococcus
low-grade fever that can last for up to a
D. Mumps virus week.
E. Neisseria meningitis 3. Joint pain: Joint pain is a common symptom
of rubella in adults
4. Headache
5. Sore throat and cough: Adults with rubella
may also experience a sore throat or cough.
6. Swollen glands: Swollen glands, especially
behind the ears and at the base of the skull.
79. During medical examination a cadet in the naval college was Objective
detected to have a painless dense ulcer 1.5x0.5 in size in his Painless ulcer is in genitalia and rectum is a key
perianal area at the 2 o’clock position. The ulcer floor feature of syphilis.
resembles” old fat”. What is the provisional diagnosis?
A. Hard syphilitic chancre of the rectum
B. Rectal fissure
C. Rectal fistula
D. Anal cancer
E. Anal crypt suppuration
80. A 32-year-old woman complains of tumorlike formation on Objective
the anterior surface of her neck that appeared 2 years ago. Thyroid tumor
Within the last 3 months the tumor has been rapidly growing. It • Thyroid adenoma: benign tumor follicles
hinders swallowing and impairs speech; the tumor causes a without vasculature. May not cause any
sensation of pressure. Objectively the skin moisture is normal, symptom.
pulse is 80/min., rhythmic, blood pressure is 130/80 mm Hg. In • Thyroid cancer:
the right lobe of the thyroid gland there is a dense lumpy node • Rapidly growing tumor
3.0x3.5 cm that moves during swallowing. Scanning image • Including papillary, follicular, medullary,
and anaplastic thyroid cancer.
shows a “cold nodule” in the thyroid gland. Make the
• Symptoms include a lump or swelling in
provisional diagnosis:
the neck, hoarseness, difficulty
A. Thyroid cancer swallowing, and pain in the neck or throat
B. Thyroid adenoma • Thyroid cyst is a fluid-filled sac that develops
C. Thyroid cyst in the thyroid gland.
D. Nodular goiter • Nodular goiter: is a condition in which the
E. Autoimmune thyroiditis thyroid gland becomes enlarged and develops
one or more nodules or lumps.
• Autoimmune thyroiditis: also known as
Hashimoto's thyroiditis, is a condition in which
the immune system attacks the thyroid gland
leading to hypothyroidism.

Thyroid adenoma
81. After a surgery for a left thigh phlegmon the Objective
disease progression was complicated by sepsis. On the Metabolic phases of sepsis, rather than the clinical phases of
7th day after the surgery there are marked signs of a sepsis, can be divided into these phases:
generalized inflammatory reaction, in blood there are 1. Catabolic Phase: This is the initial phase of sepsis, which is
signs of toxic anemia and progressing characterized by an ↑ breakdown of energy stores in the
hypoproteinemia, bilirubin levels are 40 mcmol/L, AST body, including proteins and fats; might present with
and ALT exceed the norm by 2.5 times. Oliguria anemia.
persists (700 mL of urine per day). Name the phase of 2. Stress Phase: In this phase, the body attempts to increase
sepsis progression: energy production to support the immune response.
Hormones such as cortisol and epinephrine are released, and
A. Catabolic phase
the body begins to break down muscle tissue to provide
B. Stress phase
amino acids for energy.
C. Anabolic phase 3. Anabolic Phase: This phase is characterized by a decrease in
D. Recovery phase the breakdown of energy stores and an increase in energy
E. Mixed phase production. The body begins to rebuild tissues and organs
that may have been damaged during the earlier phases of
sepsis.
4. Recovery Phase: This phase is marked by the resolution of
the metabolic response to sepsis. Energy stores are
replenished, and the body returns to its normal state.
82. A 10-year-old boy, who was outdoors in windy and cold Frostbite
weather, developed moderate pain and tingling in his fingers Frostbite is a cold-induced injury that occurs when skin
and underlying tissues freeze due to exposure to extreme
and toes. When he returned home, his parents noticed that the cold. The degrees of frostbite are:
tips of his fingers and toes were white and their sensitivity was 1. First-Degree Frostbite: This is the mildest form of
lost. As the affected areas were warming up, the fingers and toes frostbite, which affects only the outermost layer of
developed tingling and painful sensations. Skin pallor changed the skin (epidermis). Symptoms include numbness,
into redness, tingling stopped, mild itching and swelling of the tingling, and redness of the affected area. No blisters.
Can be managed by appling apandage on the
fingers appeared. Determine the frostbite degree in this child: 2. Second-Degree Frostbite: In this stage, the injury
A. Frostbite of the I degree extends beyond the epidermis and affects the deeper
B. Perniosis layers of the skin (dermis). Symptoms include the
C. Frostbite of the II degree formation of blisters, swelling, and a burning or
prickling sensation in the affected area.
D. Frostbite of the III degree
3. Third-Degree Frostbite: This is a severe form of
E. Frostbite of the IV degree frostbite, in which the injury extends deeper into the
tissues, affecting muscles, tendons, and even bones.
The affected area may appear white or blue, and the
skin may feel hard and cold to the touch. Blisters
may be present, and the affected area may feel numb.
4. Fourth-Degree Frostbite: This is the most severe
form of frostbite, in which the tissue damage is so
extensive. In some cases, amputation may be
necessary. The affected area may appear black or
dark blue, and the tissue may be completely numb.
A patient with frostbite of both feet was delivered to an
admission ward. What actions should be taken?
A. To apply a bandage, to introduce vasodilating medications
B. To administer cardiac medications
C. To put feet into hot water
D. To rub feet with snow
E. To apply an alcohol compress

A patient, aged 58, was fishing in the winter. On return home


after some time felt some pain in the feet. Consulted a doctor.
On examination: feet skin was pale, then after rewarming
became red, warm to the touch. Edema is not significant, limited
to the toes. All types of sensitivity are preserved. No blisters.
What degree of frostbite is observed?
A. I degree
B. II degree
C. III degree
D. IV degree
E. V degree
A patient with signs of general overexposure to cold presenting
with local frostbites of fingers has been delivered into an
admission room. Objectively: conscious, inert, speech is slow,
the skin of the face is cold, body temperature is 34oC, heart rate
is 68/min.What would be the actions of a doctor on call?
A. Hospitalize the patient to the surgical department
B. Hospitalize the patient to the therapeutics department
C. Hospitalize the patient to the traumatology department
D. Let the patient go home
E. Refer to a family doctor on the next day
83. A 16-year-old patient has made an appointment with an Objective
otolaryngologist. He complains of elevated body temperature Lacunar tonsillitis and follicular tonsillitis are two types of
tonsillitis, which is an infection or inflammation of the
and sore throat. Disease onset was 2 days ago, after the patient tonsils.
ate two portions of ice-cream. Pharyngoscopy shows hyperemic • Lacunar tonsillitis occurs when the small pits or crypts in
mucosa of the palatine tonsils, with purulent exudate in the the tonsils, called lacunae, become inflamed and filled
lacunae. Make the provisional diagnosis: with pus.
A. Lacunar tonsillitis • Follicular tonsillitis: occurs when the infection affects the
tonsil's follicles, which are small pockets in the tonsils
B. Follicular tonsillitis that produce white blood cells.
C. Diphtheria
D. Acute pharyngitis
E. Pseudomembranous (Vincent’s) tonsillitis
84. A 35-year-old woman complains of high body temperature Objective
and pain in the upper outer quadrant of her right buttock, which A small surgery can be performed to remove an
developed after an injection. She has been presenting with this abscess.
condition for 3 days. At the site of injection, the skin is
hyperemic; there is a painful infiltrate with an area of softening
in its center. The woman is diagnosed with a postinjection
abscess of the right buttock. What tactics should the surgeon
choose in this case?
A. Abscess incision, sanation and drainage of the cavity
B. Hospitalization, prescription of antibiotics, UHF
C. Abscess puncture, pus removal followed by application of
antiseptics
D. 10-15 minutes of low-intensity laser radiation directed at the
right buttock
E. Antipyretic agents, massage, and application of dry heat to
the right buttock
85. A 65-year-old woman on abdominal palpation presents with
a tumor in the umbilical region and above it; the tumor is 13x8 Objective
cm in size, moderately painful, nonmobile, pulsing. On Aneurysm: is an abnormal bulging or ballooning
of a weakened section of an artery. Rupture of
auscultation systolic murmur can be observed. What is the most
aneurysm is one of the most serious
likely diagnosis?
complications.
A. Abdominal aortic aneurysm
B. Gastric tumor
C. Arteriovenous aneurysm
D. Tricuspid insufficiency
E. Bicuspid insufficiency
86. A 32-year-old man complains of pain in his legs that Vasculitis
intensifies during walking, intermittent claudication, numbness 1. Buerger disease (Obliterating endarteritis):
of his toes, extremity coldness, and inability to walk more that • Risk factors: Heavy tobacco smoking
100 meters. When he sleeps, his leg usually hangs down. The history, males < 40 years old.
patient has been smoking since he was 16. He drinks alcohol in • Findings: intermittent claudication. May
excess. The left leg is colder than the right one; the skin of the lead to gangrene, autoamputation of
extremities is dry. No pulse can be detected on the pedal digits, superficial nodular phlebitis.
arteries, while pulsation of the femoral arteries is retained. What • Segmental thrombosing vasculitis with
is the most likely diagnosis? vein and nerve involvement.
A. Obliterating endarteritis 2. Immunoglobulin A vasculitis (Henoch-
B. Diabetic angiopathy Schonlein purpura):
C. Leriche syndrome (aortoiliac occlusive disease) • Most common childhood systemic
vasculitis. Often follows URI.
D. Raynaud disease
• Classic triad:
E. Deep thrombophlebitis
1. Hinge pain (arthralgias)
2. Stomach pain (abdominal pain
associated with intussusception)
3. Palpable purpura on buttocks/legs
Disease onset was acute. A child developed general weakness, Vasculitis
pain in the joints, and elevated temperature. Later these signs 1. Buerger disease (Obliterating endarteritis):
became accompanied by itching skin rash manifested as • Risk factors: Heavy tobacco smoking
erythematous spots 25 mm in size. The rash gradually turned history, males < 40 years old.
hemorrhagic. Large joints are painful and swollen; pain attacks • Findings: intermittent claudication. May
periodically occur in the paraumbilical area; there are signs of lead to gangrene, autoamputation of
intestinal hemorrhage. What is the most likely diagnosis? digits, superficial nodular phlebitis.
A. Hemorrhagic vasculitis (Henoch-Schonlein purpura) • Segmental thrombosing vasculitis with
B. Scarlet fever vein and nerve involvement.
2. Immunoglobulin A vasculitis (Henoch-
C. Hemorrhagic meningoencephalitis
Schonlein purpura):
D. Streptococcal impetigo
• Most common childhood systemic
E. Rheumatism vasculitis. Often follows URI.
• Classic triad:
1. Hinge pain (arthralgias)
2. Stomach pain (abdominal pain
associated with intussusception)
3. Palpable purpura on buttocks/legs
87. A 50-year-old patient was brought to a hospital with Objective
complaints of blood in urine. Urination is painless and Painless hematuria, smoking, age group is key
undisturbed. Macrohematuria had been observed for 3 days. findings in renal cell carcinoma.
Objectively: kidneys cannot be palpated, suprapubic area is
without alterations, external genitalia are nonpathologic. On
rectal investigation: prostate is not enlarged, painless, has Renal cell carcinoma
normal structure. Cystoscopy revealed no changes. What is the Key features:
most likely diagnosis? • Most common 1 renal malignancy.
A. Renal carcinoma • Most common in males 50–70 years old,
B. Bladder tuberculosis • ↑ incidence with tobacco smoking and obesity.
C. Varicocele Findings:
D. Dystopic kidney • Polygonal clear cells A filled with accumulated
E. Necrotic papillitis lipids and carbohydrate. Often golden-yellow
due to ↑ lipid content.
• Metastasis to lung and bone.
• Manifests with flank pain, palpable mass,
hematuria (classic triad) as well as anemia,
fever, weight loss.
88. A 59-year-old man complains of pain in his left eye and left
side of his head, significant vision impairment of the left eye, Objective
nausea, and vomiting. Visual acuity of the right eye is 1.0. Glaucoma
• Is a damage in the optic nerve due to increased
Visual acuity of the left eye is 0.03, attempts at correction bring
intraocular pressure in the eye.
no improvement. Right eye intraocular pressure - 21 mm Hg,
• Normal intraocular pressure is10 to 21 mmHg
left eye intraocular pressure 65 mm Hg. Congestive injection is
• Pathophysiology: ↑ aqueous humor = ↑
observed on the sclera of the left eye. The cornea is thick and intraocular pressure ↑ = damage optic nerve =
swollen. The anterior chamber is shallow, moist, and clear. The peripheral vision loss (peripheral retina is more
pupil is dilated and unresponsive to the light, the fundus of the susceptible to damage) = central vision loss
eye is not visible. What is the most likely diagnosis? • Types: open angle (chronic) and closed angle
A. Acute attack of glaucoma of the left eye (acute)
B. Acute iridocyclitis of the left eye
C. Stage II intraocular tumor of the left eye
D. Endophthalmitis of the left eye
E. Panophthalmitis of the left eye
Glaucoma Drugs
• M3 agonists
• Contract ciliary muscle
Related • Carbachol, pilocarpine
A man diagnosed with closed-angle glaucoma, grade IIa, of the • α2 agonists
right eye is registered for regular medical check ups. In the • Block ciliary epithelium from releasing
evening an acute glaucoma attack occurred in his right eye; an aqueous
ambulance was called. What emergency aid would be optimal in • Apraclonidine
this case? • Β blockers
A. Pilocarpine, Diacarb (Acetazolamide), lytic mixture • Block ciliary epithelium from releasing
B. Atropine eye drops aqueous.
C. Antibiotic eye drops, broad-spectrum • Timolol, betaxolol, carteolol
D. Sulfacetamide sodium eye drops • Prostaglandin analogues
E. Dexamethasone eye drops • Vasodilate the Canals of Schlemm =
increase outflow
• Bimatoprost
• Carbonic anhydrase inhibitors:
• Decrease synthesis of aqueous
• Acetazolamide
89. On the 15th day after a small trauma of the right foot, the Objective
patient developed indisposition, fatigability, irritability, Muscle spasm, tension, and muscle twitching are
headache, elevated body temperature, and sensation of common symptoms found in individuals with
constriction, tension, and twitching in the muscles of the right tetanus infection. Open wounds is one of the risk
shin. What disease can be suspected? factors for tetanus infection.
A. Tetanus
B. Anaerobic gas gangrene
C. Erysipelas
D. Acute thrombophlebitis
E. Thrombophlebitis of the popliteal artery
Clostridium Family
1. Anerobic bacteria ‫ﻻ ھواﺋﯾﺔ‬
2. Secretes Exotoxins.
3. Hate air (anerobic).
4. Kitt-Tarozzi method.
5. Neutraliztion test.
6. Spore forming.
7. Tennis racket
8. Transmission: Canned food and honey.
Clostridium botulinum:
1. Secretes botulinum exotoxin: a neurotoxin which inhibits the
release Acetylcholine =
2. No muscle contraction (flaccid paralysis ‫)ﺷﻠل ارﺗﺧﺎﺋﻲ‬.
1. No swallowing, No movement, No respiration, No eye
accommodation.
3. Note: Botox is also used in cosmetics to remove wrinkles ‫اﻟﺗﺟﻌدات‬.
Clostridium tetani:
1. Secretes tetanus exotoxin: a neurotoxin which inhibits the release
of inhibitory neurotransmitter =
1. No inhibition to the muscles =
2. Muscles spams (Spastic paralysis).
Clostridium perfringens.
1. Causes: Gas gangrene
2. Produces Homolysis in the blood.
Shock ‫ھﺑوط ﺣﺎد ﻓﻲ اﻟدورة اﻟدﻣوﯾﺔ‬
90. A patient has the second and third degree burns of the 15% Sever drop in the blood circulation (hypotension) →
Objective
of the body surface. On the 20th day after the trauma the patient No blood supply → shock)
Burns → infection → sepsis (septic
presents with sharp increase of body temperature, general Ischemia →
weakness, rapid vesicular respiration; facial features are Hypoxemia → cell death
sharpened, BP is 90/50 mm Hg, heart rate is 112/min. What 1. Cardiogenic:
complication is it? • The heart doesn’t pump the blood →
A. Sepsis • No cardiac output → Sever Hypotension.
B. Pneumonia 1. Management:
C. Acute intoxication • Dobutamine
• Promedol
D. Purulent bronchitis
2. Septic shock: Bacteraemia → toxins → massive
E. Anaerobic infection
vasodilation → shock
3. Hypovolemic shock:
• Loss of blood because of hemorrhage (> or =
30%) →
• Sever drop in blood pressure.
• Management:
• Adrenaline.
• Sodium chloride solution.
4. Anaphylactic shock:
1. Type 1 hypersensitivity.
2. Histamine → Vasodilation → sever drop in
blood pressure. (very fast)
91. 2 hours after a traffic accident a 28-yearold man in a grave Objective
condition was brought to a hospital. The patient complains of • Traffic accident → major concerns for
abdominal pain. He received a blow to the abdomen with the significant internal abdominal injuries →
steering wheel. Objective examination revealed the following: laparotomy.
the abdomen does not participate in respiration, is tense and • Laparoscopy on the other hand can be done to
acutely painful on palpation; the abdominal muscles are specific procedure in a less sever form
defensively tense, peritoneal irritation signs are positive, hepatic • Terminology:
• Lapara-: abdomen
dullness is absent. BP is 90/60 mm Hg, heart rate is 120/min.
• -otomoy: cut
What further treatment tactics should be chosen?
• -oscopy: look at
A. Laparotomy
B. Laparoscopy
C. Cold to the abdomen
D. Ultrasound investigation
E. Laparocentesis
Why laparotomy?
• Laparotomy may be performed after a traffic accident if there is concern for
significant internal abdominal injuries like we have in this case. A traffic
accident can cause a range of injuries, from minor bruises and cuts to more
serious injuries like fractures, internal bleeding, and organ damage. If there is
a suspicion of internal abdominal injuries, such as a ruptured spleen, liver, or
bowel, a laparotomy may be necessary to assess the extent of the damage and
to repair or remove any injured organs.
• During the procedure, the surgeon can directly visualize the organs inside the
abdomen and identify any injuries that may not be visible on imaging
studies, such as CT scans. The surgeon can then repair any injuries, remove
any damaged or non-functional organs, and stop any bleeding. This can be
especially important in cases where there is significant internal bleeding, as it
can quickly become life-threatening.
• In addition to being a diagnostic tool and a means of repairing injuries,
laparotomy can also help to prevent future complications. For example, if
there is a tear or perforation in the bowel, a laparotomy can allow the
surgeon to repair it and prevent a serious infection or blockage from
developing.
• Overall, the decision to perform a laparotomy after a traffic accident will
depend on the severity of the injuries and the specific needs of the individual
patient. It is a major surgical procedure that carries some risks, but it can be a
life-saving intervention in cases of significant internal abdominal injuries.
92. A 48-year-old woman has arrived to the surgical unit with Objective
wounds in her thigh. On examination the wound surface has Clostridium perfringens is an anaerobic bacteria
dirty-gray coating with unpleasant sweet smell. Wound content that cause gas gangrene wounds. Symptoms may
resembles raspberry jelly. Skin tissues around the wound are include edema, pain, gas with crepitation, foul-
glossy and turgid. Palpation reveals moderate crepitation in the smelling exudates, intense coloration of the site.
tissues. What microflora is the most likely to cause such
inflammation?
A. Anaerobic clostridial
B. Anaerobic non-clostridial
C. Streptococci
D. Staphylococci
E. Blue pus bacillus
Clostridium Family
1. Anerobic bacteria ‫ﻻ ھواﺋﯾﺔ‬
2. Secretes Exotoxins.
3. Hate air (anerobic).
4. Kitt-Tarozzi method.
5. Neutraliztion test.
6. Spore forming.
7. Tennis racket
8. Transmission: Canned food and honey.
Clostridium botulinum:
1. Secretes botulinum exotoxin: a neurotoxin which inhibits the
release Acetylcholine =
2. No muscle contraction (flaccid paralysis ‫)ﺷﻠل ارﺗﺧﺎﺋﻲ‬.
1. No swallowing, No movement, No respiration, No eye
accommodation.
3. Note: Botox is also used in cosmetics to remove wrinkles ‫اﻟﺗﺟﻌدات‬.
Clostridium tetani:
1. Secretes tetanus exotoxin: a neurotoxin which inhibits the release
of inhibitory neurotransmitter =
1. No inhibition to the muscles =
2. Muscles spams (Spastic paralysis).
Clostridium perfringens.
1. Causes: Gas gangrene
2. Produces Homolysis in the blood.
93. After a pain attack in the right subcostal area, a 58-year-old Objective
woman with overnutrition developed icteric skin and sclera, The presence of these symptoms suggests the
light colored feces, and dark urine. Her abdomen is distended patient is experiencing gallstones (cholelithiasis)
and painful on palpation in the right subcostal area. Palpation which is best diagnosed.
detects liver enlargement by 2-3 cm. Blood test: total bilirubin -
90 mcmol/L, conjugated bilirubin - 60 mcmol/L. What method
of examination will be the most informative for diagnosis
clarification?
A. Retrograde cholangiopancreatography
B. Intravenous cholegraphy
C. Infusion cholegraphy
D. Percutaneous transhepatic cholegraphy
E. US of the hepatopancreatobiliary zone

Check jaundice video for more details


94. An 11-year-old boy for a month has been presenting with Objective
increasing pain in the right femur. In the painful area there is a Ewing sarcoma is a rare and aggressive type of
nonmobile painful tumor with unclear margins. The child cancer that usually affects children and young
complains of general indisposition, weakness, increased body adults.
temperature up to 39oC. X-ray shows widened medullary cavity, • Common in patients < 15 years old.
small foci of cancellous bone destruction, and onion-like • Effects long bones like femur and pelvic flat
lamellar exfoliation of the cortical layer. What is the most likely bones.
pathology resulting in such clinical presentation? • Onion skin appearance on effected area.
• Aggressive with metastasis.
A. Ewing sarcoma
B. Osteogenic sarcoma
C. Fibrosarcoma
D. Chondrosarcoma
E. Juxtacortical sarcoma
95. A 43-year-old man complains of a protrusion in the right Objective
inguinal region, that enlarges due to strain. He has been • Hernias: is when internal organs gets out of
presenting with this condition for 6 months. Within this period place.
the protrusion has grown. Objectively in the right inguinal • Inguinal hernia: when part of abdominal cavity
region an elastic protrusion 8x5 cm is visible. On palpation it gets into scrotum or labia major.
disappears, leaving an empty space 4x4 cm between the
pedicles of the Poupart ligament. “Cough push” sign is positive
over this opening. Make the diagnosis:
A. Right-sided reducible inguinal hernia
B. Right-sided reducible femoral hernia
C. Cyst of the right spermatic cord
D. Right-sided inguinal lymphadenitis
E. Right-sided reducible arcuate line hernia
96. A 78-year-old man with a prostate adenoma underwent a Objective
herniotomy for a direct inguinal hernia. After the surgery he Anuria (little or no urination) is one of the
presents with absent urination. Enlarged urinary bladder is possible complication after herniotomy surgery. If
detectable above the patient’s pubis. What measures should be that happened, an urgent collection of urine is
taken in this case? required by a urinary catheter.
A. Bladder catheterization
B. Apply cold to the urinary bladder area
C. Prescribe processing of the postoperative wound with UHF
field
D. Prescribe proserin (neostigmine) intramuscularly
E. Prescribe antispasmodics subcutaneously
Objective
97. A 38-year-old patient has been brought by an ambulance to Dotted hemorrhages on the skin and
the surgical department with complaints of general weakness, thrombocytopenia are typical presentation of one
indisposition, black stool. On examination the patient is pale, of the platelets disorders like thrombocytopenic
there are dotted hemorrhages on the skin of his torso and purpura.
extremities. On digital investigation there are black feces on the
glove. Blood test: Hb- 108 g/L, thrombocytopenia. Anamnesis Bleeding disorders
states that a similar condition was observed 1 year ago. Make Types of bleeding disorders
the diagnosis: • Coagulation factors disorders:
A. Thrombocytopenic purpura • Deep bleeding: Joint bleeding, deep tissue
B. Hemophilia bleeding
C. Ulcerative bleeding • Disorders:
D. Rectal tumor 1. Hemophilia A, B, or C: ↑ PT
E. Nonspecific ulcerative colitis 2. Vitamin K deficiency
• Platelets disorders:
• Superficial bleeding: Mucosal bleeding,
skin bleeding, petechiae
• Disorders: Bernard-Soulier syndrome,
Glanzmann thrombasthenia, Immune
thrombocytopenia, Uremic platelet
dysfunction, Thrombocytopenic purpura.
• Mixed: von Willebrand disease and
Disseminated intravascular coagulation
98. A 30-year-old man came to the family physician. 2 months Objective
ago, he underwent a surgery for open fracture of the humerus. • Posttraumatic osteomyelitis is a bone infection
On examination the patient’s condition is satisfactory; in the that occurs as a complication from trauma,
area of the postoperative wound there is a fistula that discharges such as a fracture or surgery.
a small amount of pus; the area itself is red; fluctuation is • Findings: pain, pus, swelling, redness, and
detected. X-ray shows destruction of the humerus with warmth at the site of the injury or surgery.
sequestra. What complication did the patient develop during the
postoperative period?
A. Posttraumatic osteomyelitis
B. Hematogenous osteomyelitis
C. Wound suppuration
D. Posttraumatic phlegmon
E. Suture sinus
99. 3 hours after a trauma, a young man developed bradycardia Objective
of 46/min., anisocoria D>S, hemi-hyperreflexia S>D, • Neurological symptoms after trauma →
hemihypesthesia on the left, and a convulsive disorder. The suspected brain injury → CT scan or MRI
character of this process needs to be clarified. What method of • After a head trauma, a CT scan can detect
examination will be the most accurate for this purpose? bleeding or swelling in the brain, skull
A. Brain CT fractures, and other types of brain injuries.
B. Skull X-ray • Skull X-ray will mainly limited to skull
fracture only.
C. Electroencephalography
D. Echoencephalography
E. Lumbar puncture
100. The body of a 24-year-old woman with suspected Objective
poisoning has been found on the street. Forensic medical Forensic doctor’s job is to identify the cause of
examination was requested by an investigator during death.
examination of the site and the body. According to the Criminal
Procedure Code currently in force in Ukraine, forensic medical
examination is required when it is necessary to determine the:
A. Cause of death
B. Manner of death
C. Time of death
D. Mode of death
E. Mechanism of death
101. A 37-year-old patient complains of pain in the spinal Objective
column, reduced mobility. The condition persists for 7 years.” • Ankylosing spondylitis (AS) is a type of
Sway back” is observed, there is no movement in all spinal inflammatory arthritis that primarily affects
regions. X-ray shows” bamboo spine” vertebral column. What mainly the spine → bamboo spine (vertebral
is the most likely diagnosis? fusion).
A. Ankylosing spondyloarthritis • Associations: patients with AS can also
B. Osteochondrosis develop inflammation of the uveitis (ex,
Iridocyclitis).
C. Spondylitis deformans
• Terminology: "Ankylos” means "stiffness" or
D. Tuberculous spondylitis
"fusion," and "spondylos" means "vertebra."
E. Spondylolisthesis
Objective
• Ankylosing spondylitis (AS) is a type of
inflammatory arthritis that primarily affects
Review mainly the spine → bamboo spine (vertebral
A 40-year-old man with Bekhterev disease (ankylosing fusion).
• Associations: patients with AS can also
spondylitis) complains of elevated body temperature up to
develop inflammation of the uveitis (ex,
37.8oC, back pain and stiffness, especially observed during the
Iridocyclitis).
second half of the night. This condition has been lasting for 2 • Terminology: "Ankylos” means "stiffness" or
years. Objectively: reduced spinal mobility, painful sacroiliac "fusion," and "spondylos" means "vertebra."
joint, erythrocyte sedimentation rate - 45 mm/hour. X-ray shows
narrowing of the intervertebral disc space and of the sacroiliac
joint. What eye pathology is often associated with this type of
disease progression?
A. Iridocyclitis
B. Retinal detachment
C. Cataract
D. Optic nerve atrophy
E. Blepharitis
102. A surgery unit received a person with an incised stab Objective
wound on the upper third of the right thigh. Examination detects Arterial blood is brighter, deeper and stronger
an incised stab wound 3.0x0.5x2.0 cm in size on the inner
surface of the upper third of the right thigh. Bright-red blood
flows from deep within the wound in a pulsing stream.
Characterize this type of bleeding:
A. Arterial
B. Venous
C. Parenchimatous
D. Capillary
E. Mixed
103. A 47-year-old man developed the signs of decompensated Objective
laryngeal stenosis against the background of acute flegmonous • Tracheostomy is a surgical procedure in which
laryngitis. He presents with inspiratory dyspnea at rest, forced a hole is created in the front of the neck and a
position, cyanotic skin covered in cold sweat, tachycardia, tube is inserted into the trachea to help a
deficient pulse, and low blood pressure. What urgent treatment person breathe.
tactics should be chosen? • This may be done if a person has difficulty
A. Tracheostomy breathing due to an obstruction or narrowing of
B. Oral administration of hyposensitization substances and the airway.
broncholytics
C. Intravenous administration of dehydrating agents
D. Administration of glucocorticoid hormones
E. Oxygen therapy
104. Heart X-ray of a 31-year-old man has revealed the
following: with tightly filled opacified esophagus there is a
marginal filling defect in its middle third on the posterior wall;
the defect is 1.8x1.3 cm in size with clear oval border. Mucosal
folds are retained and envelop the defect; wall peristalsis and
elasticity are not affected. There are no complaints regarding the
condition of the patient’s alimentary canal. Make the provisional
diagnosis:
A. Esophageal tumor
B. Achalasia cardiae
C. Esophageal burns
D. Diverticulum
E. Barrett esophagus
Esophageal carcinoma Esophageal Achalasia
• Squamous cell or adenocarcinoma • What is it? failure of LES to relax due to
• Both types: ↑ risk in smokers degeneration of inhibitory neurons in the
• Presents with narrowing of the esophagus myenteric (Auerbach) plexus of esophageal
(tumor growth) and progressive dysphagia wall.
• Starts with solids • Presents with: progressive dysphagia to solids
• Progresses to liquids as tumor grows and liquids (vs obstruction—primarily solids),
• Other symptoms: retrosternal pain.
• Weight loss • Associated with: ↑ risk of esophageal cancer.
• Hematemesis • Manometry findings include uncoordinated or
absent peristalsis with ↑ LES resting pressure.
• Barium swallow shows dilated esophagus with
area of distal stenosis (“bird’s beak” A ).
• Treatment: surgery, endoscopic procedures
(eg, botulinum toxin injection).
105. A 25-year-old man was hospitalized with complaints of Acute appendicitis
pain in his lower abdomen and right lumbar area that appeared • Acute inflammation of the appendix (blue
one hour ago. Patient’s general state is moderately severe. Body arrow in ), can be due to obstruction by fecalith
temperature - 38.2oC, heart rate - 102/min. The tongue is dry. (in adults) or lymphoid hyperplasia (in
The abdomen is painful on deep palpation in the right iliac area children).
• Location: RLQ.
and in the Petit triangle. Aure-Rozanov and Gabay signs are
• Findings: Nausea, fever; may perforate →
positive. Make the provisional diagnosis:
peritonitis, rebound tenderness on exam,
A. Acute appendicitis
positive Aure-Rozanov sign.
B. Right-sided renal colic
C. Cecal tumor
D. Intestinal obstruction
E. Acute cholecystitis
106. A 45-year-old man diagnosed with acute pulmonary Objective
abscess suddenly developed sharp pain in his chest on the right • Pyopneumothorax: pus in the pleural cavity
and dyspnea up to 30/min. Examination detects facial cyanosis • Pneumothorax: air in the pleural cavity
and shallow rapid respirations. Auscultation reveals acutely • Pleuropneumonia: inflammation of lung and
weakened respiration throughout the whole right lung; pleura.
percussion reveals a vesiculotympanitic (bandbox) resonance at
the lung apex and dullness in the lower lobe. What complication
developed in this patient?
A. Pyopneumothorax
B. Pleuropneumonia
C. Pneumothorax
D. Acute mediastinitis
E. Esophageal perforation
107. A 5-year-old child was brought to the ENT department by Objective
an ambulance. The child presents with cough and difficult If a child shows symptoms of airway obstruction
respiration. From the patient’s history it is known that the child after playing with a toy, it indicates that a foreign
was playing with a toy construction set, when suddenly started object may have caused the obstruction.
coughing and developed labored breathing. Examination detects
periodical cough, labored expiration, and respiratory lag in the
left side of the child’s thorax. Auscultation: diminished
respiration on the left. Percussion: tympanitis. X-ray shows a
displacement of the mediastinal organs to the right. Make the
diagnosis:
A. A foreign body in the left bronchus, valvular
bronchostenosis
B. A foreign body in the right bronchus, valvular
bronchostenosis
C. A foreign body in the trachea
D. A foreign body in the left bronchus, complete
bronchostenosis
E. A foreign body in the right bronchus, partial bronchostenosis
108. A 30-year-old man was brought to the neurosurgical Objective
department with complaints of constant headaches, nausea, • The patient had a complication of cerebral
vomiting, fever, and weakness of the right-side limbs. abscess after surgery for otitis and mastoiditis.
Anamnesis states that one month ago the patient had a surgery • Abscess: is cavity filled with pus.
for left-sided suppurative otitis and mastoiditis. He has been
undergoing treatment in an ENT department. Approximately 2
weeks ago the temperature increased, and the patient developed
headaches. Objectively: heart rate - 98/min., BP- 140/90 mm
Hg, temperature 38.3oC. Neurologically manifested stiff neck:
bilateral Kernig’s symptom, unsteadiness during the Romberg’s
maneuver. Computer tomography of the brain revealed a three -
dimensional growth with a capsule in the left hemisphere. Make
the diagnosis:
A. Cerebral abscess
B. Echinococcus
C. Hemorrhage
D. Hydrocephalus
E. Arnold-Chiari malformation
109. The burns unit received a patient, who 6 hours ago during a
Objective
fire received flame burns. On the patient’s body there is
Decompression necrectomy is a surgical
graybrown area of necrosis that covers 3/4 of the body
procedure that involves the removal of dead or
perimeter. Occasionally there are small blisters with damaged tissue (necrosis) from a wound or injury
hemorrhagic contents and patches of shredded epidermis. What site. It is typically used to treat severe or life-
local therapy is necessary in this case? threatening infections, such as necrotizing
A. Decompression necrectomy fasciitis.
B. Chemical necrolysis
C. Blister puncture
D. Necrectomy with xenotransplantation
E. Necrectomy with dermal autograft
110. A woman in her early- to mid-thirties has lost her Objective
consciousness 3-5 minutes ago. On examination: the skin is • Clinical death: absences of pulse and
pale, no pulse over the carotid arteries, no spontaneous respiration; brain and other organs might be
respiration, pupils are dilated; the patient is nonresponsive, functioning; CPR or defibrillation is required.
presents with atony. The patient’s condition can be determined • Biological death (brain death): irreversible
as: cessation of all biological functions in the
A. Clinical death body; pupil is not responsive.
B. Natural death
C. Syncope
D. Brain death
E. Comatose state
111. A boy had a foreign body removed from under his nail Objective
plate. 3 days later he developed a sharp throbbing pain at the • Terminology:
end of his distal phalanx, which intensifies when the phalanx is • Sub: under
pressed, hyperemia of the nail fold, elevated body temperature • Unguis: nail
up to 38.5oC, and nail plate discoloration. Make the diagnosis: • Panaritium: abscess
A. Subungual panaritium • Subungual panaritium" literally means an
B. Erysipelas abscess or inflammation under the nail
C. Paronychia
D. Erysipeloid
E. Abscess
112. A 32-year-old woman complains of body weight loss Objective
despite her increased appetite, nervousness, and tremor of the Diffuse toxic goiter
extremities. Objectively: the skin is moist; the thyroid gland is (Graves' disease)
diffusely enlarged, painless, soft, and mobile. Blood test: • Most common cause of hyperthyroidism.
increased level of T3, T4, and thyroid-stimulating hormone • Pathophysiology: Thyroid-stimulating
(THS). What is the most likely diagnosis? immunoglobulin stimulates TSH receptors on:
A. Diffuse toxic goiter • Thyroid → hyperthyroidism, diffuse
B. Thyroid carcinoma goiter
• Dermal fibroblasts → pretibial
C. Autoimmune (Hashimoto’s) thyroiditis
myxoedema
D. Thyroid adenoma
• Orbital fibroblasts → Graves orbitopathy.
E. Diffuse nontoxic goiter • Lab Findings: ↑ T3, T4 and TSH
113. A 19-year-old young man complains of cough with Objective
expectoration of purulent sputum in the amount of 100 mL per • Purulent cough and dilatation are typical for
day, hemoptysis, dyspnea, increased body temperature up to chronic bronchitis.
37.8oC, general weakness, weight loss. The patient’s condition • It’s one of the four COPD diseases:
lasts for 4 years. Exacerbations occur 2-3 times a year. The Emphysema, chronic bronchitis, asthma,
patient presents with malnutrition, pale skin, cyanosis of the Bronchiectasis
lips, drumstick (clubbed) fingers. Tympanic percussion sound in
the lungs, weakened respiration, numerous various moist
crackles in the lower pulmonary segments on the left can be
observed. In blood: erythrocytes - 3.2·1012/L, leukocytes -
8.4·109/L, ESR- 56 mm/hour. On X-ray: lung fields are
emphysematous, the left pulmonary root is deformed and
dilated. What is the most likely diagnosis?
A. Multiple bronchiectasis of the left lung
B. Chronic left-sided pneumonia
C. Chronic abscess of the left lung
D. Left-sided pulmonary cystic dysplasia
E. Suppuration of the cyst in the left lung
114. A 57-year-old woman during a regular ultrasound Objective
examination presented with a space-occupying heterogeneous • Based on the description provided in the case,
lesion in the right kidney. What is the most informative method the diagnosis may be a renal mass or kidney
of renal tumor diagnostics? tumor.
A. Spiral computed tomography • CT scan is the most informative scan for
B. Excretory urography suspected kidney tumor.
C. Retrograde pyelography
D. Radioisotope renography
E. Three glass urine test
115. A 40-year-old victim of a traffic accident sustained the Objective
following injuries: closed diaphyseal femur fracture, brain Priority for the most serious injuries
concussion, multiple rib fractures, hemopneumothorax,
degloving shin injuries. What injuries require the most urgent
attention?
A. Multiple rib fractures, hemopneumothorax
B. Closed diaphyseal femur fracture
C. Brain concussion
D. Degloving shin injuries
E. All injuries are equivalent
116. At the railroad crossing a passenger train collided with a Objective
bus. In this collision 26 bus passenges died, another 18 Hospital Emergency Codes
passengers received mechanical injuries of varying severity. Code Gray: This code is typically used to indicate
Where will be professional medical aid provided for the victims a severe weather emergency, such as a tornado or
of this accident? Who will provide this aid? hurricane.
A. In medico-prophylactic institutions; general physicians and Code Triage: This code is used to indicate a mass
surgeons casualty incident, such as a major accident or
B. At the site of the accident; first-response emergency teams disaster, where a large number of patients need
immediate medical attention.
C. At the site of the accident; specialized second-response
Code MCI: MCI stands for Mass Casualty
emergency teams
Incident, and this code is used to indicate a
D. In medico-prophylactic institutions; specialized second- situation where there are more patients than the
response emergency teams hospital can typically handle.
E. In medical institutions; all listed types of healthcare workers Code Disaster: This code is used to indicate any
type of large-scale disaster or emergency, such as
a natural disaster, terrorist attack, or major
accident.
Code Red: This code is sometimes used
specifically for fire-related emergencies, such as a
fire in the hospital or a nearby building.
117. A 45-year-old man underwent a cardiac surgery one week Objective
ago. His general state has been deteriorating since then: dyspnea Pericardial friction rub, dyspnea, chest pain with
at rest, retrosternal pain that irradiates to the neck, marked irradiation to the neck is a typical finding in acute
weakness. Objectively his body temperature is hectic. His pericarditis.
cardiac borders are expanded, apical beat is weakened.
Auscultation detects pericardial friction rub. What is the most
likely diagnosis?
A. Acute pericarditis
B. Acute cardiac aneurysm
C. Myocardial infarction
D. Acute myogenic dilatation of the heart
E. Pulmonary embolism
118. A 45-year-old man was brought by an ambulance into the
Objective
emergency hospital. He complains of sudden pain in the lumbar
Symptoms of kidney stones
area, frequent painful urination, and vomiting. Examination • Severe colicky pain in the side and back, and
detects pain in the lumbar area, costovertebral angle tenderness, radiates to the lower abdomen and groin
pain on palpation of kidneys and along the ureter on the right. • Dysuria Painful urination
Urine test: proteins, fresh erythrocytes, leukocytes. Make the • Hematuria
provisional diagnosis: • Nausea and vomiting Fever and chills (if there
A. Urolithiasis, renal colic is an infection).
B. Acute pyelonephritis
C. Acute glomerulonephritis
D. Acute renal failure
E. Polycystic kidney disease
119. A newborn girl has Apgar score of 7-8 points at the 1-5
minutes after birth. During the labor there was a brief difficulty Erb-Duchenne palsy
• Erb-Duchenne palsy, is a type of upper brachial
with extraction of the shoulder girdle. After birth the baby
plexus injury that occurs during childbirth or
presents with disturbed function of the proximal segment and
trauma and effects the upper arm.
forced position of the right arm. The shoulder is rotated inwards, • How the injury happens?
the elbow is extended, the forearm is pronated, and the whole 1. Stretching of the baby's neck and
upper limb resembles an arm of a doll. What is the most likely shoulder during delivery or trauma →
clinical diagnosis in this case? 2. Damage to upper brachial plexus nerves
A. Erb-Duchenne palsy →
B. Thoracic spine trauma 3. Paralysis of the shoulder and upper arm,
C. Osteomyelitis of the right arm along with a loss of sensation in the
D. Intracranial hemorrhage affected area (hands is not involved).
E. Soft tissue injury of the right arm • Findings:
• Inward rotation of shoulder
• Arm extended and pronated
Related
A baby born after fast labour has palsy of hand muscles. Grasp
reflex is absent, as well as hand-to-mouth reflex. Hand
Klumpke palsy
sensitivity is absent. What is the most likely diagnosis?
• Erb-Duchenne palsy, is a type of lower
A. Dejerine-Klumpke palsy
brachial plexus injury that occurs during
B. Duchenne-Erb’s palsy
childbirth or trauma and effects the upper arm.
C. Total lesion of the brachial plexus
• How the injury happens?
D. Muscle paresis
1. Stretching of the baby’s arm and shoulder
E. Bernard-Horner syndrome
during delivery or trauma →
2. Damage to lower brachial plexus nerves

3. Paralysis of weakness or paralysis of the
hand and forearm, along with a loss of
sensation in the affected area.
• Findings:
• Grasping defect.
Vasculitis
120. Disease onset was acute. A child developed general
1. Buerger disease (Obliterating endarteritis):
weakness, pain in the joints, and elevated temperature. Later
• Risk factors: Heavy tobacco smoking
these signs became accompanied by itching skin rash
history, males < 40 years old.
manifested as erythematous spots 25 mm in size. The rash • Findings: intermittent claudication. May
gradually turned hemorrhagic. Large joints are painful and lead to gangrene, autoamputation of
swollen; pain attacks periodically occur in the paraumbilical digits, superficial nodular phlebitis.
area; there are signs of intestinal hemorrhage. What is the most • Segmental thrombosing vasculitis with
likely diagnosis? vein and nerve involvement.
A. Hemorrhagic vasculitis (Henoch-Schonlein purpura) 2. Immunoglobulin A vasculitis (Henoch-
B. Scarlet fever Schonlein purpura):
C. Hemorrhagic meningoencephalitis • Most common childhood systemic
D. Streptococcal impetigo vasculitis. Often follows URI.
E. Rheumatism • Classic triad:
1. Hinge pain (arthralgias)
2. Stomach pain (abdominal pain
associated with intussusception)
3. Palpable purpura on buttocks/legs
121. A 13-year-old girl for the last two weeks has been
Heart failure
complaining of dyspnea and shin and foot edemas that appear • Heart failure is inability of the heart to pump
after a physical exertion. In the morning the edemas enough blood supply.
significantly decrease. Clinical examination revealed enlarged • Edema, dyspnea and liver enlargment are key
liver and coarse systolic murmur over the heart area. Blood test features of heart failure.
and urinalysis are without changes. What is the most likely • In this question, normal blood and urine analysis
cause of edemas in this child? excluded other cuases of edema.
A. Heart failure
B. Nephrotic syndrome
C. Acute pyelonephritis
D. Angioneurotic edema
E. Hepatic cirrhosis
122. A 7-year-old boy has severe pulmonary mucoviscidosis Objective
(cystic fibrosis). He complains of dyspnea and blood Echocardiography is the primary diagnostic tool
expectoration. Objectively he presents with lagging physical utilized by cardiologists. It provides detailed
development, acrocyanosis, hepatomegaly, drumstick fingers, images of the heart's structure and function.
and nail plates resembling a” clock face”. Provisional diagnosis
of chronic pulmonary heart disease is made. What examination
would be the most informative for diagnosis confirmation?
A. Doppler echocardiography
B. Electrocardiography
C. Chest X-ray
D. Rheography of the pulmonary artery
E. Ultrasound of the liver
123. Mother of a 5-year-old child noticed on the head of her Microsporia – tinea capitis
child a round” bald” spot 3 cm in diameter. All the hairs in the • Microsporia, also known as ringworm of the scalp or
focus are broken off at the length of 5-6 mm. The day before the tinea capitis, is a fungal infection of the scalp and
child was petting a stray cat. Make the diagnosis: hair. It is caused by a group of fungi known as
A. Microsporia dermatophytes, which can infect the skin, nails, and
B. Superficial trichophytosis hair.
• Microsporia is more commonly found in children,
C. Deep trichophytosis
especially those between the ages of 3 and 7. The
D. Psoriasis
infection is usually transmitted through direct contact
E. Alopecia areata with an infected person or animal, or through contact
with contaminated objects such as combs, brushes,
hats, or towels.
• Symptoms of microsporia may include circular, scaly
patches on the scalp, broken hair shafts, and itching.
The affected areas may also be red and inflamed. In
some cases, there may be pus-filled sores or swollen
lymph nodes.
• Treatment for microsporia usually involves
antifungal medications, such as griseofulvin,
terbinafine, or itraconazole.
124. A 2-year-old child with persisting cough and subfebrile Objective
body temperature after a case of URTI developed dyspnea, • Pleurisy, also known as pleuritis, is a medical
cyanosis of the nasolabial triangle, percussion dullness and condition characterized by inflammation of the
weakened respiration in the lower lobe of the right lung, and a pleura, which is the thin tissue lining the lungs
slight mediastinal displacement to the left. What pulmonary and chest cavity, usually due to infection.
pathology is likely to cause this clinical presentation? • Symptoms include, chest pain, Shortness of
A. Pleurisy breath, and fever.
B. Emphysema
C. Pneumonia
D. Atelectasis
E. Bronchitis
Objective
The patient suffers from pleural effusion is
accumulation of fluid in the pleural cavity.
• Types of pleural effusion:
Review • Exudate: fluid is reach in cells and
A 57-year-old patient complains of dyspnea at rest. The patient proteins ( > 25g/l)
presents with orthopnea, acrocyanosis, bulging cervical veins. • Transudate: fluid is clear (protein <25 g/l)
On percussion: dull sound over the lower lung segments. On
auscultation: no respiratory sounds. Heart rate is 92/min. Right-
sided cardiac dilatation is observed. The liver is +7 cm. Shins
are swollen. Pleural effusion is suspected. What indicator would
confirm the presence of transudate in this case?
A. Total protein content in the pleural fluid below 25 g/L
B. Presence of atypical cells
C. Total protein content in the pleural fluid exceeding 30 g/L
D. Specific gravity exceeding 1015
E. Positive Rivalta’s test
125. During examination a 4-month-old child with Terminology
meningococcemia presents with acrocyanosis, cold extremities, Shock ‫ھﺑوط ﺣﺎد ﻓﻲ اﻟدورة اﻟدﻣوﯾﺔ‬
tachypnea, and thready pulse, blood pressure of 30/0 mm Hg,
anuria, and sopor. What clinical syndrome is it? Objective
A. Toxic shock syndrome Toxic shock syndrome
B. Neurotoxicosis • Toxic shock syndrome (TSS) is a rare life-
C. Exicosis threatening condition caused by certain types of
D. Encephalic syndrome bacteria, most commonly Staphylococcus aureus
E. Acute renal failure (staph) or Streptococcus pyogenes (strep).
• Symptoms: high fever, low blood pressure,
vomiting, anuria, cyanosis, diarrhea, and a rash
that looks like sunburn.
126. At night a 2-year-old child with upper respiratory tract Objective
infection suddenly developed dyspnea with labored inspiration. Stenosing laryngotracheitis (croup)
Objectively the skin is pale, perioral cyanosis and slight • What is it? Inflammation of the larynx (voice
acrocyanosis are observed. Breathing is loud, respiration rate is box) and trachea (windpipe), primarily in young
32/min. Jugular, supra- and infraclavicular fossae retract during children. It is caused by viral infections, most
breathing. Respiration is coarse on auscultation. Heart sounds commonly parainfluenza virus.
are clear and sonorous, heart rate is 120/min. What condition • Findings:
was complicated by the development of the upper respiratory • Barking cough
tract infection? • Inspiratory stridor
A. Stenosing laryngotracheitis • Narrowing of upper trachea (Steeple sign).
B. Airway foreign body • Croup: old English word "kropan", which means
C. Obstructive bronchitis "to cry out with a hoarse voice"
D. Bronchiolitis
E. Bronchial asthma
127. A 1-year-old child with a case of URTI suddenly developed Objective
noisy respirations with difficult inspiration, intercostal Stenosing laryngotracheitis (croup)
retractions, and barking cough on the 2nd night after the disease • What is it? Inflammation of the larynx (voice
onset. What is the most likely diagnosis? box) and trachea (windpipe), primarily in young
A. Stenosing laryngotracheobronchitis children. It is caused by viral infections, most
B. Acute pulmonary inflammation commonly parainfluenza virus.
C. Bronchial asthma • Findings:
D. Acute bronchitis • Barking cough
• Inspiratory stridor
E. Acute bronchiolitis
• Narrowing of upper trachea (Steeple sign).
• Croup: old English word "kropan", which means
"to cry out with a hoarse voice"
128. A 10-year-old boy with symptoms of arthritis and Objective
myocarditis was brought to a hospital. Based on clinical Types of arthritis
examination the provisional diagnosis of juvenile rheumatoid 1. Rheumatoid Arthritis:
arthritis was made. What symptom is the most contributive for • Erosion (usuration) and joint space
the diagnostics of this disease? narrowing
A. Reduced mobility of the joints in the morning 2. Osteoarthritis:
B. Regional hyperemia of the joints • Joint space narrowing
• Subchondral sclerosis
C. Affection of the large joints
D. Enlarged heart
E. Increased heart rate

Normal rheumatoid arthritis

Check the video for more details.


129. A 7-year-old girl has been twice treated with antibacterial Objective
agents for urinary tract infection. US shows no severe renal Children who suffer from repeated urinary tract
defects. The child presents with recurrence of leukocyturia and infections may have vesicoureteral reflux, a
bacteriuria, elevated body temperature up to 38.5oC, and pain in condition in which urine flows back from the
her left lumbar area. What examination should be conducted bladder to the kidneys which can be diagnosed with
first to clarify the cause of urinary infection recurrence? micturating cystourethrography.
A. Micturating cystourethrography
B. Excretory urography vesicoureteral reflux
C. Retrograde pyelography • Retrograde flow of urine from bladder toward
D. Immunogram upper urinary tract.
E. Radioisotope renography • Can be 1° due to abnormal/ insufficient insertion
of the ureter within the vesicular wall
(ureterovesical junction [UVJ]) or 2° due to
abnormally high bladder pressure resulting in
retrograde flow via the UVJ.
• ↑ risk of recurrent UTIs.
• Diagnosis: Micturating cystourethrography
130. A child is 1 year old. After solid food was introduced into Objective
the diet, within the last several months the child developed loss Patients with celiac disease should exclude glutin
of appetite, diarrhea with large amount of feces, and occasional cotaining food from their diets such as wheat, oats,
vomiting. Body temperature remains normal. Body weight is 7 bread, pasta, cereals
kg. The child is very pale, has leg edemas and extremely
distended abdomen. Feces analysis detects high levels of fatty
acids and soaps. Diagnosis of celiac disease was made and Celiac disease
gluten-free diet was prescribed. What should be excluded from Gluten-sensitive enteropathy
the diet in this case? • Definition: chronic autoimmune disorder that
affects the small intestine in response to the
A. Cereals - wheat, oats
ingestion of gluten.
B. Milk and dairy products
• Pathophysiology: gluten
C. Fruits
D. Animal protei
E. Easily digestible carbohydrates
131. A 7-year-old boy has been an inpatient for 1.5 months. He Objective
had been brought to the hospital with complaints of edemas all Proteinuria > 3.5 g/L is diagnostic for nephrotic
over his body, low urine output, and headache. Clinical syndrome.
urinalysis: proteins 7.1 g/L, leukocytes - 1-2 in the vision field,
erythrocytes - 3-4 in the vision field. During the course of
treatment, the edemas gradually dissipated, headache abated,
diuresis normalized. Daily urine proteins 3 g/L. Biochemical
blood test: total protein 43.2 g/L, urea - 5.2 mmol/L, cholesterol
- 9.2 mmol/L. What glomerulonephritis syndrome is the most
likely to be present in the patient?
A. Nephrotic
B. Nephritic
C. Isolated urinary
D. Hematuric
E. Mixed
132. A 3-month-old child with signs of rickets presents with Sings of hypocalcaemia
positive Chvostek, Trousseau, and Maslov signs. One day ago, • You need calcium in blood to suppress neurons
the parents witnessed a cyanotic attack in their child the child and relax muscles.
broke into a cold sweat, the eyes bulged, and respiratory arrest • No PTH → no calcium → no nerve inhibition
occurred. One minute later the child drew in a loud breath and → seizure / convulsions.
the child’s condition normalized again. What is the cause of the • Deficiency: Seizure → convulsions ‫ ﺗﺷﻧﺟﺎت‬and
described signs of the disease? Muscle spasm, positive Chvostek and
Trousseau signs.
A. Decrease of blood calcium levels
B. Increase of blood calcium levels
C. Decrease of blood phosphorus levels
D. Increase of blood phosphorus levels
E. Metabolic acidosis
Objective
133. A newborn with gestational age of 31 weeks presents with • Cerebrospinal fluid, which is a clear,
hypotonia and depressed consciousness. Hematocrit is 35%, colorless liquid that surrounds the brain
general cerebrospinal fluid analysis shows increased content of and spinal cord.
erythrocytes and protein, and low glucose. These data • The patient is suffering from hypertensive
correspond with the clinical presentation of: crisis ( > 180/120 mmHg).
A. Intracranial hemorrhage • Hypertensive crisis can lead to serious
B. Meningitis complication including stroke and bleeding
C. Sepsis in the brain and its layers leading to blood
in the CSF (Subarachnoid hemorrhage)
D. Anemia
E. Intrauterine infection
134. A newborn has Apgar score of 9. When should this infant
be put to the breast? Objective
Apgar score
A. In the delivery room
Newborn with an Apgar score of 9 is considered a
B. After 12 hours
good score and can be put to the breast immediately
C. After 2 hours after birth.
D. On the 2nd day
E. On the 3rd day
Apgar score
Assessment of newborn vital signs following
delivery via a 10-point scale evaluated at 1 minute
and 5 minutes. Apgar score is based on appearance,
pulse, grimace, activity, and respiration. Apgar
scores < 7 may require further evaluation. If Apgar
score remains low at later time points, there is ↑
risk the child will develop long-term neurologic
damage.
135. A 3-week-old infant developed large, flaccid vesicles with Pemphigus neonatorum
purulent contents on the skin of chest and abdomen. The Pemphigus neonatorum is a rare autoimmune
vesicles rupture quickly. Make the provisional diagnosis: disease caused by the transfer of maternal
A. Pemphigus neonatorum antibodies against a specific protein, called
B. Vesiculopustulosis desmoglein 1, to the baby during pregnancy. These
C. Toxic erythema antibodies attack the baby's skin and mucous
D. Pemphigus syphiliticus membranes, causing blistering and erosions.
E. Pseudofurunculosis Blisters can get infected resulting in formation of
purulent inflammation.
Review
A 53-year-old man complains of general weakness, loss of
Objective
Acantholytic pemphigus
appetite, and painful vesicles appearing on his skin. The disease
• Terminology:
onset occurred suddenly, after hyperinsolation one week ago. • Pemphigoid: blister or bubble
Examination detects isolated vesicles with wrinkled opercula • Acantholysis: separation of skin
and occasional painful erosions on the skin of the patient’s torso • What is it? Autoimmune disease in which
and limbs. Nikolsky sign is positive. What is the most likely immune system produces antibodies that attack
diagnosis? proteins in the skin and mucous membranes,
A. Acantholytic pemphigus leading to the formation of blisters and
B. Nonacantholytic pemphigus erosions..
C. Duhring’s disease (dermatitis herpetiformis)
D. Herpes
E. Toxicodermia
Related
10 days after birth a newborn developed a sudden fever up to
38, 1oC. Objectively: the skin in the region of navel, abdomen
and chest is erythematous; there are multiple peasized blisters
with no infiltration at the base; single bright red moist erosions
with epidermal fragments on the periphery. What is your
provisional diagnosis?
A. Epidemic pemphigus of newborn
B. Syphilitic pemphigus
C. Streptococcal impetigo
D. Vulgar impetigo
E. Atopic dermatitis
136. 10 hours after birth a child developed jaundice, hypotonia,
Objective
hyporeflexia, and moderate hepatosplenomegaly. Feces and
The infant presents with symptoms of “hemolytic
urine are of normal color. Umbilical cord blood bilirubin is 51 disease of the newborn” which can be diagnosed
mcmol/L due to unconjugated bilirubin levels. In venous blood: with coombs test.
erythrocytes - 3.5·1012/L, Нb- 140 g/L, reticulocytes - 1.5%,
bilirubin - 111 mcmol/L, conjugated - 11 mcmol/L, ALT- 40
U/L, AST30 U/L. Mother’s blood group is А(II) Rh (-), child’s
blood group is А(II) Rh (+). What laboratory test can confirm
the diagnosis?
A. Coombs test
B. Viral hepatitis markers analysis
C. Measurement of erythrocyte osmotic resistance
D. Erythrocytometry
E. Measurement of glucose 6-phosphate dehydrogenase levels
in erythrocytes
Hemolytic disease of the newborn
(erythroblastosis fetalis)
• What is it? This condition occurs when the
mother has certain antibodies in her blood
that recognize and attack the red blood cells
of the fetus. The most common cause of HDN
is Rh incompatibility between the mother and
fetus, which occurs when the mother is Rh-
negative and the fetus is Rh-positive.
• Symptoms: may include jaundice, anemia,
enlarged liver or spleen, low muscle tone, and
in severe cases, brain damage or even death.
• Diagnosis: coombos test can be used for
detection of antibodies against fetus’s blood.
• Treatment: involve phototherapy to treat
jaundice, blood transfusions to replace
destroyed red blood cells, and in severe cases,
exchange transfusions to remove the baby's
affected blood and replace it with healthy
blood.
137. A 6-month-old child on breastfeeding is hospitalized in the
inpatient department. After the child recovers, the doctor Objective
recommends the mother to start introducing solid food to the ‫واﺣده واﺣده ع اﻟﻌﯾل‬
child’s diet. What products should be introduced to the child’s
diet first?
A. Vegetable puree
B. Fermented dairy products
C. Grated apple
D. Semolina porridge
E. Buckwheat porridge
138. The 5-year-old child has been ill for 2 weeks. Cough Objective
attacks developed first and were then followed by reprises. The sever and long cough in this patient is typical
During coughing the child’s face turns red and cervical veins for pertussis infection.
bulge. The cough attacks induce vomiting. X-ray shows
intensified bronchial pattern. Blood test: leukocytes 16 · 109/L ,
lymphocytes - 72%, erythrocyte sedimentation rate - 4
mm/hour. What is the most likely diagnosis?
A. Pertussis
B. Obstructive bronchitis
C. Pneumonia
Bordetella Pertussis (Whooping cough) - 100 days cough
D. Adenovirus infection
Symptoms
E. Foreign body 1. Catarrhal stage: lasts one to two weeks and is characterized
by symptoms of an upper respiratory infection such as low-
grade fever, nasal congestion, and rhinorrhea.
2. The paroxysmal stage: lasts two to eight weeks and is
characterized by paroxysms of coughing followed by an
inspiratory whoop.
3. Convalescent stage: which may last for weeks to months and
is characterized by a subsiding cough.
139. A 3-year-old child presents with dyspnea that abates in the
sitting position, occasional loss of consciousness and seizures,
delayed physical development, cyanosis, drumstick fingers.
Echocardioscopy detects aortic dextraposition, ventricular septal
defect, pulmonary artery stenosis, and right ventricular
hypertrophy. What is the most likely diagnosis?
A. Tetrad of Fallot
B. Coarctation of the aorta
C. Transposition of the great vessels
D. Ventricular septal defect
Tetralogy of Fallot
E. Acquired valvular disease
• Pathology :
1. Right ventricular hypertrophy
2. Pulmonary infundibular stenosis (most
important determinant for prognosis)
3. Overriding aorta
4. VSD
• Symptoms:
• Pulmonary stenosis forces right-to-left
flow across VSD → RVH, “tet spells”
• Squatting (sitting): ↑ SVR, ↓ right-to-left
shunt, improves cyanosis.
140. A 15-year-old girl complains of dizziness and sensation of Somatoform autonomic dysfunction
lack of air that she develops in emotionally straining situations. • What is it? physical symptoms that cannot be
Relief occurs after she takes corvalol. Objectively: explained by any known medical condition.
hyperhidrosis and marble-like pattern of the skin of her palms • The symptoms:
• Pain, fatigue, dizziness, palpitations, and
and feet. Clinical and instrumental examination revealed no
gastrointestinal disturbances, among others.
organic changes in the central nervous, cardiovascular, and • symptoms are believed to be related to
respiratory systems. What provisional diagnosis can be made? psychological or emotional factors, rather than to
A. Somatoform autonomic dysfunction any underlying physical illness.
B. Obstructive bronchitis • Somatoform → physical symptoms .
C. Bronchial asthma • ِAutonomic dysfunction → symptoms related to ANS
D. Stenosing laryngotracheitis such as heart rate, blood pressure, and digestion.
E. Acute epiglottitis
141. A 1.5-month-old child on breastfeeding presents from birth
Objective
with daily vomiting, irregular liquid foamy feces, and Improvement of symptoms after limiting dietary
meteorism, which are resistant to antibacterial and probiotic food is typical for lactose deficiency.
therapy; no increase of body mass is observed. The child’s
condition improved, when breastmilk was substituted with”
NAN low lactose” formula. What pathology is it?
A. Lactase deficiency
B. Intestinal lambliasis (Giardiasis)
C. Infectious enteritis
D. Drug-induced enteritis
E. Functional dyspepsia
142. A 13-year-old girl for a month has been complaining of Objective
fatigability, dull pain in her right subcostal area, abdominal Hypokinetic biliary dyskinesia (HBD) is a
distension, and constipations. Abdominal palpation reveals condition in which the muscles in the bile duct do
positive Kehr, Murphy, and Ortner signs, while Desjardins and not contract properly, leading to a backup of bile
Mayo-Robson points are painless. Total bilirubin is 14.7 in the gallbladder. Bile is a fluid that is produced
mcmol/L, predominantly indirect, ALT- 20 U/L, AST- 40 U/L, in the liver and stored in the gallbladder. It is
amylase - 6.3 mmol/L. Echocholecystography shows practically released into the small intestine to aid in the
digestion of fats.
no contraction of the gallbladder. Make the provisional
diagnosis:
A. Hypokinetic biliary dyskinesia
B. Hyperkinetic biliary dyskinesia
C. Chronic pancreatitis
D. Acute pancreatitis
E. Chronic hepatitis
143. A 22-day-old infant developed subcutaneous red nodes Objective
from 1.0 to 1.5 cm in size on the scalp; later the nodes • Pseudofurunculosis is a purulent lesion of
suppurated. Temperature increased up to 37.7oC, intoxication sweat glands that develops when staphylococci
symptoms appeared, regional lymph nodes enlarged. Complete or other pathogens enter them.
blood count: anemia, leukocytosis, neutrocytosis, increased • Pseudofurunculosis mainly affects newborns
ESR. What diagnosis can be made? and children under one year old
A. Pseudofurunculosis • Clinical dermatology refers pseudofurunculosis
to a group of purulent-inflammatory skin
B. Pemphigus
diseases — pyoderma. The most common
C. Vesiculopustulosis
infectious agent in pseudofurunculosis is
D. Scalp phlegmon
Staphylococcus aureus.
E. –
Objective
Sycosis is an inflammatory reaction to hair
penetrating the skin characterized by firm papules
and pustules.
10. A young man has made an appointment with the
dermatologist. He complains of a painful facial rash in the
beard and mustache area. This condition has been
persisting for several weaks already. After shaving, the
patient’s condition aggravates. The diagnosis of sycosis is
made. What primary morphological elements can be
observed in the rash in this case?
A. Pustules, papulae
B. Nodes, nodules
C. Pustules, bumps
D. Phlyctenae, maculae
E. Maculae, nodes
144. A 10-year-old boy was brought into the hospital with
Asthma: ‫ﻣرض اﻟرﺑو‬
complaints of expiratory dyspnea, respirations are 30/min. He
• Hypersensitivity type 1: Very fast.
explains his state by a change in the weather conditions. For the • Treatment:
last 4 years the boy has been registered for regular check-ups 1. Salbutamol: is the drug of choice for asthma.
due to his diagnosis of third degree persistent bronchial asthma. • Selective Β 2 -adrenergic receptors.
To provide emergency aid for this child, first he needs to be • MOA: bronchodilatation = More air
given: 2. Ephedrine:
A. Salbutamol or short-acting β2-agonists 1. hydrochloride: activates alpha and beta.
B. Dexamethasone 2. Alpha and beta are the receptor for
C. Adrenaline sympathetic
D. Euphylline (Aminophylline) 3. Beta present in lungs: 👍 brochodilation
E. Claritin (Loratadine) for asthma.
4. Alpha present in vessels: 👎
(vasoconstriction for hypertention).
3. Isadrin (isoproterenol):
1. MOA: activates all beta = Good for lung
beta 2 👍 and bad for heart (beta 1) 👎
4. Ipratropium:
1. Blcok M = no parasympathetic
145. A 3-year-old child has been brought to a hospital
with complaints of pain in the legs, fever, and loss of
appetite. Objectively: pale skin and mucosa,
hemorrhagic rash. Lymph nodes are enlarged, painless,
dense and elastic, not matted together. Bones, joints,
and abdomen are painful. The liver and spleen are
enlarged. Hemogram: Hb- 88 g/L, color index - 1.3,
platelets - 80 · 109/L, leukocytes - 25.8·109/L,
lymphoblasts - 70%, ESR- 52 mm/hour. Make the
provisional diagnosis:
A. Acute leukemia
B. Thrombocytopenic purpura
C. Acute rheumatic fever
D. Infectious mononucleosis
E. Hemorrhagic vasculitis (Henoch-Schonlein
purpura)
146. During an outdoors school event in hot weather, a 10-year- Objective
old girl lost her consciousness. Body temperature - 36.7oC. • Syncope: temporary loss of consciousness and
Objectively her skin is pale and cold to touch, her pupils are posture caused by a temporary decrease in blood
dilated. Blood pressure - 90/50 mm Hg. Heart rate - 58/min. flow and oxygen supply to the brain. fainting or
What pathology occurred in this case? passing out.
A. Syncope • Causes: including low blood pressure,
B. Sympathicotonic collapse dehydration, heart problems, neurological
C. Paralytic collapse disorders, and certain medications.
D. Sunstroke • It can also be triggered by emotional stress,
E. - pain, or sudden changes in posture.
• Sympathicotonic collapse: sudden drop in blood
pressure and syncope related to overactivity of
sympathetic nervous system.
• Weakness or paralysis: paralysis or weakness
from injury.
• Sunstroke: prolonged exposure to high
temperature → high body temperature (above
103°F or 39.4°C) → symptoms: confusion,
dizziness, headache, nausea, vomiting, rapid
heart rate, and rapid breathing. In severe cases,
sunstroke can lead to seizures, coma, or even
death
147. A 13-year-old girl has 30% of excessive body mass, she
Objective
started to gain weight at the age of 3. She has a family history of
• Alimentary Constitutive Obesity: excessive
obesity. Her height and sexual development are normal for her appetite → overeating and a lack of physical
age. The appetite is excessive. She complains of periodical activity.
headaches. Blood pressure - 120/80 mm Hg. Subcutaneous fat is • Hypothalamic Obesity: hypothalamus damage
evenly distributed, she has no stretch marks. There is juvenile → dysregulation of satiety or hanger centres →
acne on her face. What type of obesity is it? excessive appetite and obesity.
A. Alimentary constitutive obesity • Adrenal Obesity: overproduction of cortisol
B. Hypothalamic obesity (Cushing's syndrome) → excessive appetite and
C. Adrenal obesity obesity.
D. Hypothalamic syndrome of puberty • Hypothyroid Obesity: underactive thyroid gland
E. Hypothyroid obesity → slow metabolism and weight gain
• Hypothalamic Syndrome of Puberty: rare
genetic disorder called Prader-Willi syndrome
→ insatiable hunger and a slow metabolism. It
typically presents in childhood → severe
obesity.
148. An 8-year-old girl complains of frequent painful urination Urinary tract infection
in small amounts and urinary incontinence. The signs have been • Cystitis
present for 2 days already. She explains her disease by • Infection of bladder – Lower urinary tract
overexposure to cold. Costovertebral angle tenderness is absent. • Symptoms: dysuria (pain with urination),
Complete blood count is without pathologies. Urine test: frequency (going a lot), urgency (always feel like
you must go), and suprapubic pain.
leukocytes - 20-30 in the vision field, erythrocytes - 40-50 in the
• Pyelonephritis
vision field, unchanged, bacteriuria. What is the most likely
• Infection of kidney – Upper urinary tract
diagnosis?
• Symptoms: systemic symptoms (fever, chills),
A. Cystitis
flank pain, and CVA tenderness.
B. Vulvitis • Most infections ascend: Urethra → Cystitis →
C. Pyelonephritis Pyelonephritis
D. Glomerulonephritis • Diagnosis:
E. Urolithiasis • Urinalysis: cloudy urine, leukocyte esterase
(produced by WBCs in urine) • Nitrites
>10WBC/hp
• Culture: bacteriological inoculation >100,000 CFUs
149. A pregnant woman is 28 years old. Anamnesis: accelerated
Cervical insufficiency
labor complicated by the II-degree cervical rupture. The
(isthmo-cervical insufficiency)
following two pregnancies resulted in spontaneous abortions at
• Definition: is a condition in which the cervix
the terms of 12 and 14 weeks. On mirror examination: the
begins to dilate or open too early during
uterine cervix is scarred from previous ruptures at 9 and 3 pregnancy, often leading to complications such
hours, the cervical canal is gaping. On vaginal examination: the as miscarriage in 2nd-trimester pregnancy loss.
cervix is 2 cm long, the external orifice is open 1 cm wide, the • Ultrasound findings after 15 weeks:
internal orifice is half-open; the uterus is enlarged to the 12th • Cervical shortening to ≤ 2.5 cm
week of pregnancy, soft, mobile, painless, the appendages are • Cervical dilation
without changes. What diagnosis can be made? • Protrusion of fetal membranes into the
A. Isthmico-cervical insufficiency, habitual noncarrying of cervical canal.
pregnancy • Treatment: Cervical cerclage is reinforcement
B. Threatened spontaneous abortion of the cervical ring with nonabsorbable suture
C. Incipient abortion, habitual noncarrying of pregnancy material.
D. Cervical hysteromyoma, habitual noncarrying of pregnancy
E. Cervical pregnancy, 12 weeks
150. On the day 4 after the cesarean section a woman developed
Objective
fever with body temperature up to 39oC and abdominal pain. • Peritonitis is a serious medical condition that
Pulse - 104/min. She vomited twice. The patient is sluggish, her occurs when there is inflammation and infection
tongue is dry and has gray coating. The abdomen is distended. of the peritoneum.
Signs of peritoneal irritation are positive in all segments. • Etiology: perforation in the gastrointestinal tract
Peristalsis cannot be auscultated. No passage of gas occurs. (such as a ruptured appendix or a perforated
Uterine fundus is located at the level of the navel. The uterus is ulcer), a traumatic injury to the abdomen, or a
painful on palpation. The discharge is moderate and contains complication of abdominal surgery ex,
blood and pus. What is the most likely diagnosis? caesarean section
A. Diffuse peritonitis • Findings: abdominal pain or tenderness,
B. Metroendometritis abdominal distension, fever and chills, nausea
C. Progressive thrombophlebitis and vomiting, anorexia, gastrointestinal
D. Pelvic peritonitis dysfunction manifesting as diarrhea or
constipation, fatigue or weakness, tachycardia,
E. Parametritis
and hypotension.
151. A 58-year-old woman came to the gynecological clinic. Endometrial carcinoma
She complains of bloody discharge from her genital tracts. (uterine corpus cancer)
Menopause is 8 years. Gynecological examination: the uterus is Menopausal female, abnormal uterine bleeding,
slightly enlarged, dense to touch, with limited mobility; the uterine enlargement is typical findings in
uterine appendages cannot be detected; parametrium is free. endometrial carcinoma (uterine corpus cancer)
Fractional curettage of the uterine cavity yields a significant
amount of medullary substance in the scrape. What is the most
likely diagnosis?
A. Uterine corpus cancer
B. Adenomyosis
C. Chorioepithelioma
D. Uterine cervix cancer
E. Hormone-producing ovarian tumor
152. A 48-year-old woman complains of disturbed menstrual Objective
cycle: her periods last for 7-9 days and are excessively profuse • Climacteric syndrome is an old name for
throughout the last half-year. She notes occasional hot flashes in menopause Age > 45 ‫ﺳن اﻟﯾﺄس ﻋﻧد اﻟﻧﺳﺎء‬.
her head, insomnia, irritability, and headaches. Her skin is of • Findings:
normal color. Blood pressure - 150/90 mm Hg, pulse - 90/min., • Hot flashes (very common).
rhythmic. The abdomen is soft and painless. Bimanual • Disturbed menstrual cycle.
examination shows no uterine enlargement, the appendages • Mood changes, including irritability,
cannot be detected. The vaginal fornices are free. What is the depression, and anxiety
most likely diagnosis? • Insomnia or sleep disturbances
A. Climacteric syndrome • Decreased libido or sexual desire
B. Premenstrual syndrome • Urinary incontinence or increased
urgency to urinate
C. Adrenogenital syndrome
• Joint and muscle pain
D. Stein-Leventhal syndrome (polycystic ovary syndrome)
• Weight gain or difficulty losing weight
E. Uterine myoma
• Memory problems or difficulty
concentrating
153. A 30-year-old multigravida has been in labour for 18 hours. Objective
2 hours ago the pushing stage began. Fetal heart rate is clear, • This patient is diagnosed with uterine inertia
rhythmic, 136/min. Vaginal examination reveals complete (incomplete uterine contractions).
cervical dilatation, the fetal head in the pelvic outlet plane. • Some possible options that the obstetrician
Sagittal suture is in line with obstetric conjugate, the occipital may consider include:
fontanel is near the pubis. The patient has been diagnosed with • Outlet forceps delivery is a technique that
primary uterine inertia. What is the further tactics of labor may be used in certain cases during
management? childbirth when the baby's head is visible
at the vaginal opening, but delivery may
A. Outlet forceps
be difficult.
B. Labour stimulation
• Oxytocin augmentation: administering
C. Cesarean section oxytocin (a hormone that stimulates
D. Skin-head Ivanov’s forceps uterine contractions)
E. Vacuum extraction of the fetus • Cesarean delivery: If the primary uterine
inertia is severe a cesarean delivery may
be necessary to safely deliver the baby.
154. A woman is 40 weeks pregnant. The fetus is in the Objective
longitudinal lie and cephalic presentation. Pelvic size: 26-29- • The patient is diagnosed prolonged labor
3120. Expected weight of the fetus is 4800 gram. The labor (dystocia) → prolonged strong contractions (12
contractions has been lasting for 12 hours, within the last 2 hours), large sized baby (4800) → risk of
hours they were extremely painful, the parturient woman is uterine rupture.
anxious. The waters broke 4 hours ago. On external examination • Cesarean delivery may be necessary to safely
the contraction ring is located 2 finger widths above the navel, deliver the large sized baby.
Henkel-Vasten sign is positive. Fetal heart rate is 160/min.,
muffled. On internal examination the uterine cervix is fully
open, the head is engaged and pressed to the entrance into the
lesser pelvis. What is the most likely diagnosis?
A. Threatened uterine rupture
B. Complete uterine rupture
C. Hyperactive uterine contractions
D. Abruption of the normally positioned placenta
E. Anatomically contracted pelvis
155. A 23-year-old woman came to the gynecological clinic. Objective
She complains of pain, itching, and burning in her vulva, Genital herpes infection
general weakness, indisposition, elevated body temperature up • Symptoms: Painful penile, vulvar, or cervical
to 37.2oC, and headache. On examination in the vulva there are vesicles and ulcers with bilateral tender
multiple vesicles up to 2-3 mm in diameter with clear contents inguinal lymphadenopathy; can cause systemic
against the background of hyperemia and mucosal edema. Make symptoms such as fever, headache, myalgia
the provisional diagnosis: • Causes: HSV-2, less commonly HSV-1
A. Genital herpes infection • Treatment: acyclovir and ibuprofen.
B. Primary syphilis
C. Papillomavirus infection
D. Vulvar cancer
E. Cytomegalovirus infection
156. A woman with the pregnancy term of 8 weeks complains of Objective
elevated temperature up to 37.6oC, skin rash that can be • Rubella infection in the 1st trimester can cause
characterized as macular exanthema, enlargement of posterior serious congenital malformations including,
cervical and occipital lymph nodes, small amount of bloody such as deafness, blindness, heart defects, and
discharge from the genital tracts. She was examined by the intellectual disability.
infectious diseases specialist and diagnosed with rubella. What • In some countries, rubella infection during the
tactics should the obstetrician-gynecologist choose? first trimester of pregnancy can be considered
A. Abortion as a justifiable reason for abortion.
B. Prescription of antibacterial therapy
C. Prescription of antiviral therapy
D. Treatment of incipient abortion
E. Prescription of hemostatic therapy
157. A 16-year-old girl has primary amenorrhea, no pubic hair Objective
growth, normally developed mammary glands; her genotype is Testicular feminization syndrome
46 ХY; uterus and vagina are absent. What is your diagnosis? (androgen insensitivity syndrome)
A. Testicular feminization syndrome • 46, XY male presents with female appearing
B. Mayer-Rokitansky-Kuster-Hauser syndrome characteristics.
C. Cushing syndrome • Pathogenesis:
D. Sheehan syndrome • Defect in androgen receptor →
E. Cushing disease • No effect of male sex hormone androgen.
• Female-appearing genetic male (46,XY).
• Findings:
• Female external genitalia with scant
axillary and pubic hair, rudimentary
vagina;
• Uterus and fallopian tubes absent due to
persistence of anti-Müllerian hormone
from testes.
• Normal functioning testes (often found in
labia majora; surgically removed to
prevent malignancy).
• Lab: ↑ testosterone, Estrogen, LH (vs sex
chromosome disorders).
158. A 46-year-old woman came to the maternity clinic with Objective
complaints of moderate blood discharge from the vagina, which Abnormal uterine bleeding
developed after the menstruation delay of 1.5 months. On (Dysfunctional uterine bleeding)
vaginal examination: the cervix is clean; the uterus is not • Abnormal bleeding from the uterus that is not
enlarged, mobile, painless; appendages without changes. Make related to a specific menstrual period or
the diagnosis: pregnancy.
A. Dysfunctional uterine bleeding • Causes: hormonal imbalances, structural
B. Adenomyosis abnormalities, medications, or medical
conditions such as polyps or fibroids.
C. Ectopic pregnancy
D. Submucous uterine myoma
E. Cancer of the uterine body
159. A 25-year-old woman was brought into the gynecological Objective
department with profuse bloody discharge from her genital Presence of fragments of the fertilized ovum is a
tracts. She is 12 weeks pregnant, the pregnancy is planned. sign of abortion.
Within the last 3 days she was experiencing pains in her lower
abdomen that eventually started resembling cramps, she
developed bleeding. Her skin is pale, pulse - 88/min., blood
pressure - 100/60 mm Hg, body temperature - 36.8oC. Vaginal
examination: the uterus size corresponds with 11 weeks of
pregnancy, the cervical canal allows inserting 1 finger and
contains fragments of the fertilized ovum, the discharge is
bloody and profuse. What is the most likely diagnosis?
A. 12-week pregnancy, spontaneous abortion in progress
B. 12-week pregnancy, threatened spontaneous abortion
C. Disturbed menstrual cycle, hyperpolymenorrhea
D. Disturbed menstrual cycle, amenorrhea
E. Full-term pregnancy, term labor
160. A 30-year-old woman came to the gynecological Objective
department. She complains of sharp pain in her lower abdomen • Patient presentation of fever, lower abdominal
and temperature of 38.8oC. She has a history of extramarital pain, previous abortion, vaginal purulent
sexual activity and 2 artificial abortions. On gynecological discharge are typical for infection in the
examination the uterus is unchanged. The appendages are reproductive tract.
bilaterally enlarged and painful. Profuse purulent discharge is • To clarify the diagnosis, further testing may be
being produced from the vagina. What examination needs to be needed, such as culture (bacteriologic analysis)
conducted to clarify the diagnosis? of the discharge to identify the specific
infectious agent (ex, Chlamydia and
A. Bacteriological and bacterioscopic analysis
Gonorrhea)
B. Hysteroscopy
C. Curettage of the uterine cavity
D. Colposcopy
E. Laparoscopy
161. It is the 3rd day after the first normal term labor; the infant Objective
is rooming-in with the mother and is on breastfeeding. Physiological involution is a normal decrease of
Objectively: the mother’s general condition is satisfactory. size of uterus to the normal size after deliver of
Temperature is 36.4oC, heart rate is 80/min., BP is 120/80 mm baby; takes 6 weeks to complete.
Hg. Mammary glands are soft and painless; lactation is
moderate, unrestricted milk flow. The uterus is dense, the
uterine fundus is located 3 finger widths below the navel.
Lochia are sanguino-serous, moderate in volume. Assess the
dynamics of uterine involution:
A. Physiological involution
B. Subinvolution
C. Lochiometra
D. Pathologic involution
E. Hematometra
162. A 28-year-old woman complaining of irregular Objective
menstruations and infertility came to the gynecological clinic. Polycystic ovary syndrome
Menstruations occur since the age of 15, irregular, with delays Presence of multiple small cysts in the ovaries,
up to 2 months. On examination she presents with marked which can lead to hormonal imbalances and a
hirsutism and excessive body weight. On vaginal examination range of symptoms, including irregular menstrual
the uterus is reduced in size and painless. The ovaries on the periods, infertility, acne, excessive hair growth,
both sides are dense and enlarged. Ultrasound shows and obesity.
microcystic changes in the ovaries, the ovaries are 5х4 cm and
4.5х4 cm in size with dense ovarian capsule. Basal body
temperature is monophasic. What is the most likely diagnosis?
A. Polycystic ovary syndrome
B. Krukenberg tumor
C. Endometrioid cysts
D. Bilateral adnexitis
E. Ovarian carcinoma
163. An 18-year-old girl was brought into the gynecology Objective
department with complaints of elevated body temperature up to Torsion of ovarian tumor pedicle
37.8oC, sharp pain in her lower abdomen, more intense on the (adnexal torsion)
right, and difficult defecation. Vaginal examination detected a • Adnexal torsion is an emergency condition in
painful dense elastic formation 5x6 cm in the area of her right which the ovary, fallopian tube, or other
ovary. Pregnancy test is negative. What is the most likely structures near the ovary become twisted,
diagnosis? cutting off the blood supply to the affected
A. Torsion of ovarian tumor pedicle area.
• Severe pain, nausea, vomiting, and necrosis.
B. Ectopic pregnancy
• Associated with adnexal mass.
C. Appendicitis
D. Ovarian cyst rupture
E. Ovarian apoplexy
164. A 26-year-old woman presents with amenorrhea. Objective
10 months ago, she gave birth for a second time. In her Sheehan syndrome
early postpartum period, she developed a massive Pathogenesis and symptoms:
hypotonic hemorrhage. No breastfeeding. Lately she • Pregnancy →
has been presenting with loss of weight, loss of hair, • Pregnancy-induced pituitary growth →
and indisposition. Gynecological examination revealed • postpartum hemorrhage after delivery →
atrophy of the external genitals, the uterus is • ischemic infarct of pituitary→
abnormally small, no uterine appendages can be • Hypopituitarism → failure to lactate,
amenorrhea, cold intolerance
detected. What is the most likely diagnosis?
A. Sheehan syndrome (postpartum pituitary gland
necrosis) Hypopituitarism
B. Physiological amenorrhea
C. Suspected progressing ectopic pregnancy
D. Stein-Leventhal syndrome (polycystic ovary
syndrome)
E. Galactorrhea-amenorrhea syndrome
165. A 45-year-old woman came to the maternity clinic with Objective
complaints of periodical pains in her mammary glands that start Fibrocystic mastopathy
1 day before menstruation and stop after the menstruation • Age: 20–50 years old.
begins. Palpation of the mammary glands detects diffuse nodes • Premenstrual breast pain
predominantly in the upper outer quadrants. What is the most • Fibrotic lumps
likely diagnosis?
A. Fibrocystic mastopathy
B. Breast cancer
C. Mastitis
D. Hyperprolactinemia
E. Breast cyst
166. A 14-year-old girl came to the general practitioner with
Objective
complaints of weakness, loss of appetite, headache, rapid
Profuse menstruation →
fatigability. Her last menstruation was profuse and lasted for 14
Loss of blood cells →
days after the previous delay of 2 months. Objectively: the skin Posthemorrhagic anemia.
is pale, heart rate is 90/min., BP is 110/70 mm Hg, Hb is 88 g/L.
Rectal examination: the uterus and its appendages are without
changes, no discharge from the genital tracts. What
complication occurred in the patient?
A. Posthemorrhagic anemia
B. Somatoform autonomic dysfunction of hypotonic type
C. Migraine
D. Gastritis
E. Dysmenorrhea
167. A 22-year-old postparturient woman on the 12th day after Objective
the normal childbirth informs of elevated body temperature up Lactational mastitis
to 39oC for the last 3 days and pain in her right mammary gland. Breast feeding → increased risk of infection →
The right mammary gland is enlarged, hot to touch, tense, Lactational mastitis.
hyperemic, and painful. Palpation reveals there a dense
infiltration 8x8 cm with a fluctuation in its center. What is the
most likely diagnosis?
A. Postpartum period, day 12. Right-sided infiltrative-purulent
mastitis
B. Postpartum period, day 12. Right-sided serous mastitis
C. Postpartum period, day 12. Right-sided gangrenous mastitis
D. Postpartum period, day 12. Right-sided phlegmonous
mastitis
E. Postpartum period, day 12. Right-sided lactostasis
Preeclampsia
168. A 35-year-old pregnant woman with degree 1 • A pregnancy complication characterized by:
essential hypertension, developed edemas and headache 1. Hypertension
at the 33 week of her pregnancy. Objectively her general 2. Proteinuria (edema)
condition is satisfactory, blood pressure - 160/100 mm 3. organ damage (liver and kidneys)
Hg, normal uterine tone. Fetal heart rate is 140/min., • Usually occurs 3rd trimester
rhythmic. She was diagnosed with daily proteinuria - 4
Classification
g/L, daily diuresis - 1100 mL. Creatinine - 80 mcmol/L,
1. Mild preeclampsia:
urea - 7 mmol/L, platelets - 100 · 109/L. What • BP: 140/90 mmHg
complication of pregnancy occurred? • Proteinuria: of 300 mg/day.
A. Moderate preeclampsia 2. Moderate preeclampsia:
B. Severe preeclampsia • BP: 160/110 mmHg –
C. Mild preeclampsia • Proteinuria: of 2 grams mg/day.
D. Hypertensive crisis • Signs of organ dysfunction: elevated liver enzymes,
E. Renal failure thrombocytopenia (low platelet count), or renal
insufficiency (reduced kidney function).
3. Severe preeclampsia:
• BP: a bove 160/110 mmHg.
• Proteinuria: of 5 grams mg/day.
• Signs of severe organ dysfunction: eclampsia
(seizures), pulmonary edema, HELLP Syndrome.
169. A 24-year-old pregnant woman on her 37th week of
Biophysical profile (BPP)
pregnancy has been brought to the maternity obstetric service
is a prenatal test used to evaluate the health of the fetus
with complaints of weak fetal movements. Fetal heartbeats are by ultrasound. The test evaluates five fetal parameters:
95/min. On vaginal examination the uterine cervix is tilted 1. Fetal heart rate
backwards, 2 cm long, external orifice allows inserting a 2. Fetal breathing movements
fingertip. Biophysical profile of the fetus equals 4 points. What 3. Fetal movements
tactics of pregnancy management should be chosen? 4. Amniotic fluid volume
A. Urgent delivery via a cesarean section 5. Fetal muscle tone
B. Treatment of placental dysfunction and repeated analysis of • Each parameter is given a score of 0 or 2 points,
the fetal biophysical profile on the next day depending on whether it is present or absent. The
C. Doppler measurement of blood velocity in the umbilical maximum score is 10 points.
artery • A normal score > 8 while a score 4 or below may
D. Urgent preparation of the uterine cervix for delivery indicate fetal distress syndrome and urgent delivery
might be required.
E. Treatment of fetal distress; if ineffective, then elective
cesarean section on the next day
Objective
Apgar score
Assessment of newborn vital signs following
delivery via a 10-point scale evaluated at 1 minute
Review and 5 minutes. Apgar score is based on appearance,
A newborn has Apgar score of 9. When should this infant be put pulse, grimace, activity, and respiration. Apgar
to the breast? scores < 7 may require further evaluation. If Apgar
A. In the delivery room score remains low at later time points, there is ↑
risk the child will develop long-term neurologic
B. After 12 hours
damage.
C. After 2 hours
D. On the 2nd day
E. On the 3rd day
170. During regular preventive gynecological examination a 30- Objective
year-old woman was detected to have dark blue punctulated Colposcopy is a device used to examine the
”perforations” on the vaginal portion of the uterine cervix. The cervix, vagina, and vulva for any signs of
doctor suspects endometriosis of the vaginal portion of the abnormal cells or other abnormalities.
uterine cervix. What investigation method would be most
informative for diagnosis confirmation?
A. Colposcopy, target biopsy of the cervix
B. US of the lesser pelvis
C. Hysteroscopy
D. Curettage of the uterine cavity
E. Hormone testing
171. A parturient woman is 30 years old, stage I of the labor is
ongoing. The fetus is in the cephalic presentation. Auscultation
of the fetal heart sounds detects bradycardia. Evaluation of Objective
cardiotocogram yielded the following data: decrease of basal Signs of fetal distress → urgent cesarean delivery
heart rate down to 90/min., variability - monotonous (2 and • ↓ Heart rate; normal is 120-160 bpm
• Lack of variability in the fetal heart rate
less); late decelerations with amplitude of 50/min. Make the
pattern. A normal fetal heart rate pattern should
diagnosis and choose the obstetrical tactics necessary in this
have moderate variability, meaning that there
case: should be fluctuations in the heart rate of at
A. Fetal distress. Urgent cesarean section delivery least 6-25 beats per minute. monotonous refers
B. Fetal distress. Vacuum extraction delivery to a lack of variability in the fetal heart rate
C. Normal condition of the fetus. Vaginal birth pattern
D. Fetal distress. Stimulation of uterine contractions • Late decelerations: a type of abnormal fetal
E. Fetal distress. Forceps delivery heart rate pattern that occur in response to
uterine contractions. They are characterized by
a slow and gradual decrease in the fetal heart
rate that begins after the onset of the
contraction and returns to baseline after the
contraction ends. Late decelerations may
indicate that the fetus is not receiving enough
oxygen during contractions and can be a sign
of fetal distress.
172. A 27-year-old woman complains of foul-smelling discharge Objective
from her genital tracts, pain in her lower abdomen, and elevated • Postabortal endometritis
temperature. The complaints arose 2 days ago. She has a history • An infection of the lining of the uterus that
of surgical abortion at the term of 8 weeks one week ago. Mirror occurs after an abortion. The condition is
examination: the uterine cervix is clear, external orifice usually caused by bacteria entering the uterus
produces foulsmelling discharge. Vaginal examination: the during the abortion procedure.
uterus lies in anteflexion, is mobile, painful, and slightly • Symptoms of postabortal endometritis may
enlarged. The appendages are without changes. Make the include fever, pelvic pain, foul-smelling
vaginal discharge, and heavy bleeding.
provisional diagnosis:
• If left untreated, postabortal endometritis can
A. Postabortal endometritis
lead to serious complications, such as pelvic
B. Enterocolitis abscesses, sepsis, and infertility.
C. Appendicitis
D. Acute respiratory disease
E. Salpingoophoritis
173. A 17-year-old girl has made an appointment with the • Objective
doctor. She plans to begin her sex life. No signs of Human papillomavirus (HPV) is a sexually
gynecological pathology were detected. In the family history the transmitted virus that can cause various types of
patient’s grandmother had cervical cancer. The patient was cancer in the female reproductive tract. The
consulted about the maintenance of her reproductive health. following are the most common types of HPV-
What recommendation will be the most helpful for prevention related cancer in women:
of invasive cervical cancer? 1. Cervical cancer
A. Vaccination against human papillomavirus (HPV) 2. Vulvar cancer
3. Vaginal cancer
B. Vitamins, calcium, omega-3
4. Anal cancer
C. Immunomodulators
D. Antiviral and antibacterial drugs
E. Timely treatment of sexually transmitted diseases
174. A 38-year-old woman works in flax processing, she dries Objective
flax. She came to the hospital complaining of difficult • Byssinosis, also known as "brown lung
breathing, constricting sensation in her chest, and cough attacks. disease" or "Monday fever," is a type of
These signs appear on the first day of her working week and occupational lung disease caused by the
gradually diminish on the following days. What respiratory inhalation of cotton, flax, or hemp dust or other
disease is likely in this case? textile fibers. The condition is commonly seen
A. Byssinosis in textile workers who are exposed to these
B. Silicosis fibers on a regular basis.
• Byssinosis can cause a range of respiratory
C. Allergic rhinopharyngitis
symptoms, including coughing, wheezing,
D. Bronchial asthma
shortness of breath, and chest tightness. These
E. Asthmatic bronchitis symptoms typically occur at the beginning of
the workweek (hence the term "Monday
fever"), as the workers are exposed to fresh
cotton dust after a period of rest over the
weekend.
175. A district doctor has diagnosed one of his patients with Objective
dysentery. What accounting document reflects this type of Dysentery infection can be an urgent report
morbidity? because it can lead to severe dehydration,
A. Urgent report electrolyte imbalances, and other complications
B. Statistical report that can be life-threatening if not treated promptly.
C. Report on a major non-epidemic disease Additionally, dysentery can spread quickly in
D. Certificate of temporary disability crowded or unsanitary conditions, making it a
E. Control card of a patient registered for regular check-ups public health concern.
Medical Reports and Documentation

• Urgent report: This refers to a report that requires immediate attention and
action due to its urgency or criticality
• Statistical report: This refers to a report that presents statistical data related to a
particular subject or topic. It may include information such as the number of
patients treated for a particular disease, the frequency of a particular symptom,
or the success rate of a particular treatment.
• Report on a major non-epidemic disease: This refers to a report that provides
information about a significant disease that is not spreading rapidly or in
epidemic form. It may include details about the disease's symptoms, causes, and
treatment options.
• Certificate of temporary disability: This refers to a document that certifies a
person's temporary disability due to an illness or injury. It may include
information such as the date of onset, expected duration, and the nature of the
disability.
• Control card of a patient registered for regular check-ups: This refers to a
document or card that contains information about a patient's medical history and
scheduled check-ups. It may include details about the patient's medical
condition, medications, and recommended treatments, as well as the dates of
their next scheduled appointments.
176. A 39-year-old man, a battery attendant, suddenly developed Objective
weakness, loss of appetite, nonlocalized colicky abdominal Highlighted symptoms are typical for lead
pains, and nausea. Objectively his skin is gray; there is a pink- poisoning (Saturnism).
gray stripe on his gums; the stomach is soft and sharply painful.
Blood test detected erythrocytes with basophilic stippling and
anemia. The patient has a history of peptic ulcer disease of the
stomach. Constipation tends to occur every 3-4 days. What is
the most likely provisional diagnosis?
A. Saturnism (lead poisosning)
B. Acute appendicitis
C. Perforation of gastric ulcer
D. Acute cholecystitis
E. Chronic alcoholism
177. A 9-month-old infant presents with delayed tooth eruption Objective
and fontanel closure, weakness, and excessive sweating. What Vitamin D is important for tooth and bone
type of hypovitaminosis is the most likely in this child? mineralization.
A. Hypovitaminosis D
B. Hypovitaminosis C
C. Hypovitaminosis B1
D. Hypovitaminosis B6
E. Hypovitaminosis A

Check Full explaition of vitamins


178. A 10-year-old girl exhibits high level of physical Objective
development ( + 3σ), her body length increased by 10 cm within The girl resents with advanced physical
a year (which is double the norm for her age group), the number development which is an endocrine disorder.
of permanent teeth corresponds with the age norm (20), the
development of her secondary sex characteristics is three years
ahead of her age (Ма, Р, Ах, Menarche). Development rate
ahead of her biological age can occur due to:
A. Endocrine disorders
B. Acceleration
C. Certain components of her diet
D. Sports training
E. Deficient hygienic education
179. Employees work in conditions of high dust concentration. Objective
Certain chemical (silicon dioxide content) and physical • Dispersion refers to the extent to which
properties of dust aerosols contribute to the development of particles are distributed throughout the air.
occupational dust-induced diseases. What is the main physical • Aerosol (dust) particles such as silicon dioxide
property of dust aerosols? can have high dispersion levels.
A. Dispersion
B. Magnetization
C. Electric charge
D. Solubility
E. Ionization
180. In the factory cafeteria there was an outbreak of food Objective
poisoning. Clinical presentation indicates staphylococcal Staphylococcal GIT infection → GIT sample →
etiology of this disease. 15 people are sick. To confirm the Vomit mass or stool.
diagnosis of food poisoning, samples need to be sent to the
laboratory. What samples should be obtained for analysis?
A. Vomit masses
B. Blood for hemoculture
C. Blood (complete blood count)
D. Urine
E. Saliva
181. In April during the medical examination of various Objective
population groups, 27% of individuals presented with low Gingival bleeding → Vitamin C deficiency
working ability and rapid fatigability. The following symptoms
were observed in the affected individuals: swollen friable
gingiva that bleeds when pressed, hyperkeratosis follicularis not
accompanied by skin dryness. These symptoms most likely
result from the following pathology:
A. C-hypovitaminosis
B. Parodontosis
C. A-hypovitaminosis
D. B1-hypovitaminosis
E. Polyhypovitaminosis

Check Full explaition of vitamins


182. Increased general morbidity of the local population is Objective
observed in the area near a factory, where atmosphere is being Long exposure to sulfurous gas can have several
intensively polluted with sulfurous gas. What effect does negative effects on the human body.
polluted air have on human body in this case?
A. Chronic nonspecific
B. Acute specific
C. Acute nonspecific
D. Chronic specific
E. Selective
• Exposure to high levels of sulfurous gas can have several negative effects on the human
body. Sulfurous gas is a common air pollutant that is released from industrial processes
such as the combustion of fossil fuels and industrial activities like metal smelting.
• The health effects of sulfurous gas exposure can range from mild to severe, depending on
the duration and intensity of exposure. In this case, where the local population is
experiencing increased general morbidity, the following health effects may be observed:
• Respiratory problems: Exposure to high levels of sulfurous gas can irritate the respiratory
system, causing coughing, wheezing, and shortness of breath. It can also exacerbate
existing respiratory conditions like asthma and chronic obstructive pulmonary disease
(COPD).
• Cardiovascular problems: Sulfurous gas exposure can cause inflammation of the blood
vessels, which can increase the risk of cardiovascular disease, including heart attacks and
strokes.
• Neurological problems: Exposure to high levels of sulfurous gas can affect the nervous
system, causing headaches, dizziness, and fatigue.
• Eye irritation: Sulfurous gas exposure can cause eye irritation, including redness, itching,
and tearing.
• Overall, exposure to sulfurous gas can have a significant negative impact on human health,
particularly in areas with high levels of air pollution. It is essential to reduce emissions and
take measures to improve air quality to protect public health.
183. During medical examination of high and middle school Objective
students, the doctors were assessing correlation between Development of secondary sex characteristics is
biological and calendar age of the school students based on the one of the biological feature that can be assessed
following criteria: height growth rate per year, ossification of in this age group.
the carpal bones, the number of permanent teeth. What
additional development criterion should be assessed at this age?
A. Development of secondary sex characteristics
B. Body mass
C. Chest circumference
D. Vital capacity of lungs
E. Hand strength
184. During analysis of morbidity in the city, it was determined Objective
that age structure of population is different in each district. The age-standardized would represent what the
What statistical method allows to exclude this factor, so that it morbidity rates would be in each district as if they
would not skew the morbidity data? had the same age distribution as the standard
A. Standardization population.
B. Wilcoxon signed-rank test
C. Correlation-regression analysis
D. Dynamic time series analysis
E. Analysis of average values
185. Clinical statistical investigation was performed to Objective
determine effectiveness of a new pharmacological preparation Student's t-distribution can also be used to
for patients with ischemic heart disease. What parametric test estimate the reliability of the results of a clinical
(coefficient) can be used to estimate the reliability of the statistical investigation on the effectiveness of a
results? new pharmacological preparation for patients with
A. Student’s t-distribution ischemic heart disease.
B. Sign test
C. Matching factor
D. Wilcoxon signed-rank test
E. Kolmogorov-Smirnov test
186. In a rural health care area there is an increasing cervical Objective
cancer morbidity observed. The decision is made to conduct a • Targeted screening is a medical examination
medical examination of the women living in this locality. What approach that focuses on a specific population
type of medical examination is it? or group of people who are at high risk for a
A. Target particular disease or condition. For example,
B. Preliminary woman group is at risk of developing cervical
C. Regular cancer → Targeted screening
D. Complex • In contrast, a general population screening
examination is offered to all individuals in a
E. Screening
certain population, regardless of their risk
status for a particular disease or condition.
General population screening can be useful for
detecting common conditions, such as high
blood pressure or high cholesterol, but it may
not be as effective in identifying individuals
who are at high risk for specific diseases, such
as cervical cancer.
187. In the process of hiring, a prospective employee has Objective
undergone preventive medical examination and was declared fit A preliminary examination is an initial assessment
to work in this manufacturing environment. What type of that is conducted before a more comprehensive
preventive medical examination was it? examination or investigation.
A. Preliminary
B. Scheduled
C. Periodical
D. Specific
E. Comprehensive
188. On laboratory investigation of a pork sample there is 1 Objective
dead trichinella detected in 24 sections. This meat should be: • No dead trichinella is allowed.
A. Sent for technical disposal • Normal in EU countries is 30 larvae per gram
B. Allowed for sale with no restrictions of meat.
C. Processed and sold through public catering network
D. Processed for boiled sausage production
E. Frozen until the temperature of -10oC is reached in the deep
layers, with subsequent exposure to cold for 15 days

Related
During meat testing Trichinella was detected in diaphragm crura
in one of the two muscular tissue samples. What tactics should a
doctor choose regarding thismeat?
A. Technolgical disposal
B. Incineration
C. Boiling under 1,5 atmosphere
D. Preservation in 10% salt solution
E. Freezing under -12oC
189. To assess the effectiveness of medical technologies and Objective
determine the power and direction of their effect on the public • Correlation coefficient can be used to measure
health indicators, the research was conducted to study the the strength of the relationship between two
immunization rate of children and measles incidence rate by variables.
district. What method of statistical analysis should be applied in • If the correlation coefficient is positive, it
this case? would indicate that as the immunization rate of
A. Calculation of correlation coefficient children increases, the measles incidence rate
B. Calculation of morbidity index among the nonvaccinated decreases.
C. Calculation of matching factor
D. Calculation of standardized ratio
E. Calculation of statistical significance of the difference
between two estimates
190. Having studied the relationship between the distance from Objective
villages to the local outpatient clinics and frequency of visits to • The Pearson correlation coefficient is a way to
the clinics among the rural population of this area, it was measure the strength of the relationship
determined that the rank correlation coefficient in this case between two variables.
equals -0.9. How can this relationship be characterized? • For example, the relationship between two
A. Strong inverse relationship variables, like height and weight or studying
B. Strong direct relationship and grades.
• The closer the number is to 1, the stronger the
C. Moderate inverse relationship
relationship.
D. Moderate direct relationship
• If it's close to 0, then there isn't much of a
E. -
relationship at all.
• An inverse relationship means that as one
variable (in this case, the distance from
villages to the local outpatient clinics)
increases, the other variable (frequency of
visits to the clinics) decreases.
191. In the inpatient gynecological unit within a year 6500 Objective
women underwent treatment. They spent there a total of 102000 • Bed-days: number of days a hospital bed is
bed-days. What indicator of the gynecological unit work can be occupied by a patient.
calculated based on these data? • Average length of inpatient: total number of
A. Average length of inpatient stay bed-days / total number of patients.
B. Average bed occupancy rate per year • 102000/ 6500 = 15
C. Number of beds by hospital department • On average each woman stays 15 days.
D. Bed turnover rate
E. Planned bed occupancy rate per year
Average length of inpatient stay and bed-days
• Bed-days refer to the number of days a hospital bed is occupied by a patient. For example,
if a patient is admitted to a hospital on Monday and discharged on Wednesday, that would
count as three bed-days: one for Monday, one for Tuesday, and one for Wednesday. Bed-
days are an important measure of hospital utilization, as they reflect the number of patients
requiring hospital care and the amount of resources needed to provide that care.
• The average length of inpatient stay is a commonly used indicator to evaluate the
performance of a hospital unit. It is calculated by dividing the total number of bed-days by
the total number of patients who underwent treatment during a given period.
• In this case, the inpatient gynecological unit treated 6,500 women and they spent a total of
102,000 bed-days. Therefore, the average length of inpatient stay can be calculated as:
• Average length of inpatient stay = Total number of bed-days / Total number of patients
Average length of inpatient stay = 102,000 bed-days / 6,500 patients Average length of
inpatient stay = 15.69 days
• This means that on average, each woman who underwent treatment in the inpatient
gynecological unit stayed for about 15.69 days.
• By calculating the average length of inpatient stay, hospital administrators can assess the
efficiency and effectiveness of the unit, and identify areas where improvements can be
made. For example, if the average length of inpatient stay is high, it may indicate that
patients are not being discharged in a timely manner, or that there are delays in the
provision of care or services.
192. A middle school teacher with 4-year long record of work Objective
was issued a medical certificate for pregnancy and childbirth In Ukrainian law, the maternity leave requires the
leave. What amount of pay will she receive for the duration of employer to pay 100% of average salary.
her leave in this case?
A. 100% of average salary
B. 50% of average salary
C. 70% of average salary
D. 60% of average salary
E. 80% of average salary
193. In the air of the feed kitchen at the poultry factory, at the Objective
area where formula feed is being mixed, the dust concentration • Aspergillus is an allergenic fungi found in soul
reaches 200 mg/m3. Air microflora is represented predominantly and air.
by Asperqillus and Mucor fungi. What effect determines • Symptoms of Aspergillus allergy includes
pathogenic properties of the dust? sneezing, coughing, wheezing, shortness of
A. Allergenic breath, and chest tightness
B. Teratogenic
C. Mutagenic
D. Fibrogenic
E. Toxic
Vitamin B1(thiamine/ TPP)
194. During regular medical examination a lyceum 1. Function:
student presents with signs of cheilitis that manifests • Is the cofactor for transferases enzymes.
as epithelial maceration in the area of lip seal. The • Cofactor for pyruvate dehydrogenase / decarboxylase.
2. Deficiency:
lips are brightred, with single vertical cracks covered 1. Wernicke-Korsakoff syndrome (Common with alcoholics)
with brown-red scabs. These clinical signs are most 2. Beriberi: Muscle weakness and mental disability.
likely caused by insufficient content of the following Vitamin B2 (riboflavin)
in the diet: Function: Component of FAD.
A. Riboflavin Deficiency: Cheilosis and Corneal vascularization.
Vitamin B3 (Niacin / Nicotinamide/ PP)
B. Ascorbic acid
1. Function: Makes the cofactor NAD.
C. Retinol 2. Deficiency: Pellagra (4D disease): diarrhea, dementia, symmetric
D. Thiamine dermatitis and death.
E. Calciferol 3. Note: Hartnup disease causes less absorption of tryptophan which
means B3 deficiency.
Biotin B7
• Function:
1. Acetyl-CoA carboxylase
2. Pyruvate carboxylase
3. Propionyl-CoA carboxylase
• Deficiency: Alopecia
• Causes of deficiency:
• Excessive consumption of raw egg contains avidin → binds
biotin in the intestinal lumen → inhibition of biotin resorption.
195. A 30-year-old woman made an appointment with the Objective
family doctor for scheduled vaccination of her 2-year-old child. • Primary healthcare:
What type of healthcare provides such medical services? • Primary healthcare refers to the first point
A. Primary healthcare of contact that individuals have with the
B. Emergency aid healthcare system, where they receive a
C. Secondary healthcare wide range of basic healthcare services
D. Tertiary healthcare such as preventive care, health education,
E. Palliative care routine check-ups, and vaccinations.
• Primary healthcare is often provided by
general practitioners and family doctors.
• Secondary healthcare: specialized ex, internal
medicine.
• Tertiary healthcare: highly specialized ex,
neurosurgery and cancer treatment.
• Palliative care: improving the quality of life
and relieving the suffering of individuals who
are living with serious or life-limiting illnesses.
196. Human body receives from the atmosphere a number of Objective
chemicals. What type of action results in the combined effect • Synergism: combined effect that is greater than
that is less than the sum of isolated effects of these chemicals on the sum of their individual effects. 1+1 >2
the body? • Ex, penicillin and streptomycin → more
A. Antagonism effective bacterial killing. Nicotine and
B. Potentiation caffeine → increased alertness
C. Isolated action • Antagonism: combined effect of two chemicals
D. Synergistic action is less than the sum of their individual effects.
E. Complex action • Calcium channel blockers and beta
blockers are used to treat high blood
pressure. When used together, they can
have an antagonistic effect, reducing the
effectiveness of both drugs.
197. Clinical trials have proved the” Lipoflavon” drug to be Objective
effective for treatment of unstable angina pectoris in the control Single (simple) blind study: participants or the
group and experimental group of patients. Neither patients nor researchers are unaware of who is receiving the
researchers knew who belonged to which group. Name this type treatment or the placebo.
of study: Double blind stud: neither the participants nor the
A. Double blind study researchers know who is receiving the treatment
B. Simple blind study or the placebo.
C. Triple-blind study
D. Total-blind study
E. Multicenter study

You might also like